SSC GD 6 March 2019 Shift-I Previous Year Paper

SSC GD 6th March 2019 Shift-1 

Reasoning 

Instructions 

For the following questions answer them individually 

Q. 1 Which of the following answer figures will complete the given figure ? 

Answer: (B) 

 

Q. 2 Four brothers P, Q, R and S go to a hotel. They book four hotel rooms numbered 301, 302, 303 and 304. Sum of room numbers of P and R is 603 and the sum of room numbers of R and S is 604. What is the sum of room numbers of P, Q and R? 

(A) 907 

(B) 904 

(C) 903 

(D) 905 

Answer: (A) 

 

Q. 3 Four words have been given out of which three are alike in some manner, while one is different. Choose the odd one. 

(A) Bhagavad Gita 

(B) Bible 

(C) Quran 

(D) Jewish 

Answer: (D) 

 

Q. 4 Find the missing number from the below options. 

(A) 400 

(B) 458 

(C) 456 

(D) 440 

Answer: (B) 

 

Q. 5 Select the related word pair from the given alternatives. 

Teeth : Cut :: ……. : ……… 

(A) Legs : Walk 

(B) Broom : Fly 

(C) Tubelight : Heat 

(D) Food : Smell 

Answer: (A) 

 

Q. 6 In a certain code, TIM is coded as RGK, then how is PAINE coded in the same way? 

(A) NGYCL 

(B) NGYLC 

(C) NYGCL 

(D) NYGLC 

Answer: (D) 

 

Q. 7 Choose the option which completes the given figure series. 

Answer: (B) 

 

Q. 8 Select the correct option that will fill in the blank and complete the series. 

AB, BA, ABC, CBA, ABCD, ……… 

(A) DCBA 

(B) CABD 

(C) ACBD 

(D) BACD 

Answer: (A) 

 

Q. 9 Select the correct option that will fill in the blank and complete the series. 2, 7, 5, 10, 8, 13, …….. 

(A) 15 

(B) 11 

(C) 38 

(D)

Answer: (B) 

 

Q. 10 Choose the option that most closely resembles the mirror image ofthe given figure when mirror is placed at right side. 

Answer: (A) 

 

Q. 11 Select the related word pair from the given alternatives. 

Celsius : Temperature :: ……….. : ………. 

(A) Kilometer : Distance 

(B) Bank note : Office 

(C) Meter : Water 

(D) Bulb : Electric current 

Answer: (A) 

 

Q. 12 Select the option that is related to the third term in the same wayas the second term is related to the first term. 

ROSEBERRY : SEBERRYRO :: BEAUTIFUL : ? 

(A) TIFULBEAU 

(B) EAUTIFULB 

(C) AUTIFULBE 

(D) UTIFULBE(A) 

Answer: (C) 

 

Q. 13 Four letter clusters have been given out of which three are alike in some manner, while one is different. Choose the odd one. 

(A) STaY 

(B) TRaY 

(C) AWaY 

(D) CLaY 

Answer: (C) 

 

Q. 14 A piece of paper is folded and cut as shown below in the Q. figures. From the given answer figures, indicate how it will appear when opened. 

Answer: (C) 

 

Q. 15 Select the correct option that will fill in the blank and complete the series. 

0.3, 0.8, 2.3, 4.8, ………. 

(A) 7.5 

(B) 7.3 

(C) 6.5 

(D) 8.3 

Answer: (D) 

 

Q. 16 The statements below are followed by conclusions labeled I, II, III and IV. Assuming that the information in the statements is true, even if it appears to be at variance with generally established facts, decide which conclusion(s) logically and definitely follow(s) from the information given in the statements. 

Statements: 

All Iris are Cornea. 

Some Cornea are Lens. 

All Lens are Retina. 

Conclusions: 

I. Some Retina are Cornea 

II. Some Retina are Iris 

III. Some Lens are Iris 

IV. Some Cornea are Retina 

(A) Only I and IV follow 

(B) Only I and II follow 

(C) None follows 

(D) Only III and IV follow 

Answer: (A) 

 

Q. 17 The statements below are followed by two conclusions labeled I and II. Assuming that the information in the statements is true. even if it appears to be at variance with generally established facts, decide which conclusion(s) logically and definitely follow(s) from the information given in the statements. 

Statements: 

Some shirts are socks. 

No sock is red. 

Conclusion: 

I. Some socks are shirts. 

II. No shirt is red. 

(A) Only conclusion II follows 

(B) Only conclusion I follows 

(C) Either conclusion I or II follows 

(D) Neither conclusion I nor II follows 

Answer: (B) 

 

Q. 18 If PT is written as 8-10, how will NLR be written? 

(A) 7-6-12 

(B) 8-6-11 

(C) 7-6-8 

(D) 7-6-9 

Answer: (D) 

 

Q. 19 Select the option that is related to the Third term in the same way as the second term is related to the first term. 

10 : 30 :: 68 : ? 

(A) 120 

(B) 100 

(C) 112 

(D) 130 

Answer: (D) 

 

Q. 20 Four options have been given out of which three are alike in some manner, while one is different. Choose the odd one. 

(A) 6388 

(B) 3568 

(C) 1238 

(D) 2348 

Answer: (C) 

 

Q. 21 A, M, P, D, Q, R, W and are sitting around a circle facing at the centre. D is fourth to the left of A whois third to the right of M.P is third to the left of Q whois third to the left of M.R is third to the right ofW whois second to the right of B. 

In which of the following pairs is the second person sitting to the immediate right of the first person? 

(A) A(B) 

(B) Q(B) 

(C) RM 

(D) M(D) 

Answer: (A) 

 

Q. 22 Four boys A, B, C, D and four girls P,Q, R. S are playing merry go round in such a waythat they face the centre and form a circle. No two girls or no two boys are next to each other. A is to the immediate left of R, who is opposite to Q. P and Q have only C between them.P is opposite to S. who is to the immediate right of B. 

If B is the only person between R and S, then whois opposite B? 

(A) (A) 

(B)

(C) (C) 

(D) (D) 

Answer: (C) 

 

Q. 23 Choose the option figure in which the problem figure is hidden/embedded. 

Answer: (A) 

 

Q. 24 Select the correct combination of mathematical signs to replace ‘* signs and to balance the given equation: 

530 ∗ 2 ∗ 3 ∗ 5 ∗ 800 

(A) ÷ + − = 

(B) ÷ − − = 

(C) ÷ × + = 

(D) + + × = 

Answer: (C) 

 

Q. 25 Study the diagram carefully and answer the given questions. 

How many painters are neither doctors nor players? 

(A)

(B)

(C) 10 

(D)

Answer: (B) 

General knowledge 

Instructions 

For the following questions answer them individually 

 

Q. 26 Minimum Support Price for 2018-19 for Cotton of the Medium Staple variety is set at ……… 

(A) ₹2,897 per quintal 

(B) ₹1,950 per quintal 

(C) ₹5,150 per quintal 

(D) ₹2,430 per quintal 

Answer: (C) 

 

Q. 27 Who is the author of the, ‘Byomkesh Bakshi Stories’? 

(A) Sharadindu Bandyopadhyay 

(B) Ritwik Ghatak 

(C) Satyajit Ray 

(D) David Davidar 

Answer: (A) 

 

Q. 28 India defeated the team from which country in the final to win the under 19 Cricket World Cup, 2018? 

(A) England 

(B) Australia 

(C) South Africa 

(D) Pakistan 

Answer: (B) 

 

Q. 29 Which freedom fighters were hanged for the Kakori conspiracy case? 

(A) Chittaranjan Das and Vasudev Balwant Phadke 

(B) Krishnaji Gopal Karve and BaghaJatin 

(C) Ram Prasad Bismil and Ashfaqulla Khan 

(D) Khudiram Bose and Chandra Shekhar Azad 

Answer: (C) 

 

Q. 30 The relation between the period of a simple pendulum ‘T’ and its length ‘L’ is …………..(‘g’ is acceleration due to gravity) 

(A) T = 2πLg

(B) T = 4πgL

(C) T = 4πLg

(D) T = 2πgL

Answer: (A) 

 

Q. 31 The best examples of which script are found in the rock-cut edicts of Asoka? 

(A) Grantha 

(B) Brahmi 

(C) Kadamba 

(D) Sharada 

Answer: (B) 

 

Q. 32 Which tribal festival in Chhattisgarh is dedicated to goddess Kesharpal Kesharpalin Devi, one of the tribal deities? 

(A) Sarhul 

(B) Madai 

(C) Puli Kali 

(D) Minjar 

Answer: (B) 

 

Q. 33 The atmosphere of Mars is mostly composed of ……….. 

(A) Ammonia 

(B) Carbon dioxide 

(C) Oxygen 

(D) Hydrogen 

Answer: (B) 

 

Q. 34 Who was the first and the most important king of the Rashtrakuta dynasty of Karnataka who defeated the Chalukyas of Badami? 

(A) Dantidurga 

(B) Harihara I 

(C) Uttama 

(D) Bindusara 

Answer: (A) 

 

Q. 35 In which amendment to the constitution were the Bodo, Dogri, Santhali and Maithli addedto the list of recognised languages? 

(A) 73rd Constitution Amendment Act of 2003 

(B) 92nd Constitution Amendment Act of 2003 

(C) 103rd Constitution Amendment Act, 2018 

(D) 101st Constitution Amendment Act, 2016 

Answer: (B) 

 

Q. 36 Which element is placed in the 5th Period and II(A) Group Of The periodic table? 

(A) Lead (At no 82) 

(B) Tellurium (At no 52) 

(C) Barium (At no 56) 

(D) Strontium (At no 38) 

Answer: (D) 

 

Q. 37 Which of the following rivers is NOT a tributary of the Krishna? 

(A) Venna 

(B) Betwa 

(C) Bhima 

(D) Koyna 

Answer: (B) 

 

Q. 38 Which Maratha warrior died in the successful defence of the Sinhagad fort fighting against the Mughal army? 

(A) Chimaji Appa 

(B) Baji Prabhu Deshpande 

(C) Tanaji Malusare 

(D) Baji Pasalkar 

Answer: (C) 

 

Q. 39 The prefix ‘ …….. ‘ is equal to a factor of 1015 

(A) atto 

(B) peta 

(C) femto 

(D) exa 

Answer: (B) 

 

Q. 40 The concept of the Freedom of trade in India’s Constitution is adopted from the Constitution of which country? 

(A) United Kingdom 

(B) United States 

(C) Australia 

(D) Ireland 

Answer: (C) 

 

Q. 41 Mutual Funds asset base in India stood at ………. at end of November 2018. 

(A) ₹31 lakh crore 

(B) ₹47 lakh crore 

(C) ₹24 lakh crore 

(D) ₹19 lakh crore 

Answer: (C) 

 

Q. 42 The ………. are the triangular muscles of the shoulder. 

(A) Masseter 

(B) Achilles tendon 

(C) Deltoids 

(D) Sartorius 

Answer: (C) 

 

Q. 43 The Elephant is a symbol of which Indian political party? 

(A) National Congress Party 

(B) Communist Party of India 

(C) Bahujan Samaj Party 

(D) Rashtriya Janata Dal 

Answer: (C) 

 

Q. 44 Who won the Miss World title in 1997? 

(A) Reita Faria 

(B) Diana Hayden 

(C) Tara Anne Fonseca 

(D) Zeenat Aman 

Answer: (B) 

 

Q. 45 Which Indian won the Gold in Women’s freestyle wrestling, 50 kg, at the 2018 Asian Games? 

(A) Vinesh Phogat 

(B) Babita Kumari 

(C) Sakshi Malik 

(D) Kavita Devi 

Answer: (A) 

 

Q. 46 The Bhangra dance form is popular in which state? 

(A) Puducherry 

(B) Maharashtra 

(C) Punjab 

(D) Rajasthan 

Answer: (C) 

 

Q. 47 ………. alloy consists of nickel, copper and zinc. 

(A) Solder 

(B) German silver 

(C) Bell Metal 

(D) Rose Metal 

Answer: (B) 

 

Q. 48 All of the following siblings have won the Dadasaheb Phalke Award, EXCEPT …….. 

(A) Lata Mangeshkar and Asha Bhosle 

(B) (B) R Chopra and Yash Chopra 

(C) Raj Kapoor and Shashi Kapoor 

(D) Mahesh Bhatt and Vikram Bhatt 

Answer: (D) 

 

Q. 49 The Zoji La Pass is in which mountain range? 

(A) Zanskar Range 

(B) Ladakh Range 

(C) East Karakoram Range 

(D) Dhauladhar Range 

Answer: (A) 

 

Q. 50 The Union Budget 2018 allocated ₹ 5,750 crore to ………. 

(A) Operation Green 

(B) the Ujjwala Yojana 

(C) the Saubhagya Yojna 

(D) the National Livelihood Mission 

Answer: (D) 

Quant 

Instructions 

Study the following table carefully to answer the questions given below. 

Q. 51 The total marks obtained by ‘A’ in all the subjects together is: 

(A) 260 

(B) 240 

(C) 228 

(D) 242 

Answer: (B) 

Explanation: 

Marks obtained by (A) in Maths = 88 

Marks obtained by (A) in Science = 86 

Marks obtained by (A) in English = 66 

Therefore, Total marks obtained by (A) = 88+86+66 = 240 marks. 

 

Q. 52 What is the difference between A’s and C’s total marks in the exam? 

(A) 15 

(B) 13 

(C) 16 

(D) 22 

Answer: (C) 

Explanation: 

Total Marks obtained by (A) = 88+86+66 = 240 marks 

Total marks obtained by (C) = 72+68+84 = 224 marks 

Required difference = 240-224 = 16 marks. 

Instructions 

For the following questions answer them individually 

 

Q. 53 If a man cuts the grass of a lawn in T minutes, what part of the lawn can he cut in 30 minutes? 

(A) T/30 

(B) T-30 

(C) 30-T 

(D) 30/T 

Answer: (D) 

 

Q. 54 The base area of a cuboid is 34 sq cm. and height is 3.5 cm. What is the volume of cuboid? 

(A) 125 cm3 

(B) 97 cm3 

(C) 119 cm3 

(D) 108 cm3 

Answer: C

 

Q. 55 The value ofis: 

(A) 16/21

(B) 17/21

(C) 15/21

(D) 9/21

Answer: (A) 

 

Q. 56 Sixty percent of the rats included in a particular experiment were female rats. If some of the rats died during the experiment and 30% of rats died were male, what was the ratio of the death rate among the male rats to the death rate among the female rats? 

(A) 3 : 4 

(B) 9 : 14 

(C) 9 : 16 

(D) 6 : 7 

Answer: (B) 

 

Q. 57 What should come in place of Q. mark (?) in the following questions? 

(140 + 80 × 2 − 24) ÷ (80 + 20 × 3 − 40) =? 

(A) 276 

(B) -27.6 

(C) 2.76 

(D) 0.276 

Answer: (C) 

 

Q. 58 What is the fourth proportional to 3, 7 and 15? 

(A) 30 

(B) 45 

(C) 35 

(D) 25 

Answer: (C) 

 

Q. 59 A sum of ₹ 5000 is invested for two years under compound interest at 10% p.a, interest being compounded annually. The interest earned (in ₹) is: 

(A) 2100 

(B) 1000 

(C) 1050 

(D) 500 

Answer: (C) 

 

Q. 60 A train passes the platform in 18 seconds and a man standing on the platform in 8 seconds. If the length of the train is 100 metres, the length of the platform is: 

(A) 125 m 

(B) 140 m 

(C) 150 m 

(D) 130 m 

Answer: (A) 

 

Q. 61 John travels ⅔rd of a distance by bike and remaining 6 km on foot. How many km does he travels in the whole journey? 

(A) 28km 

(B) 18km 

(C) 26km 

(D) 14km 

Answer: (B) 

 

Q. 62 Virat Kohli has certain batting average of 9 innings. In the tenth innings he scored 106 runs there by increasing his average by 6 runs. His new batting average is: 

(A) 48 

(B) 56 

(C) 52 

(D) 46 

Answer: (C) 

Instructions 

Study the following table carefully to answer the questions given below. 

Q. 63 The average marks obtained by all the students in Maths is: 

(A) 79 

(B) 87 

(C) 81 

(D) 75 

Answer: (D) 

Instructions 

For the following questions answer them individually 

 

Q. 64 If the length of a rectangle is increased in the ratio 4 : 5 and its breadth is decreased in the ratio 3 : 2, then its area will be decreased in the ratio ………… 

(A) 2 : 1 

(B) 4 : 3 

(C) 10 : 3 

(D) 6 : 5 

Answer: (D) 

 

Q. 65 If a/b = ⅔, then the value of (5a3 – 2a2b) : (3ab2 – b3) is:  

(A) 16:27 

(B) 32:29 

(C) 34:19 

(D) 27:16 

Answer: (A) 

 

Q. 66 In what proportion must a grocer mix one kind of bajra at ₹ 45 per kg with another at ₹ 32 per kg in order that by selling the mixture at ₹ 48 per kg he may make a profit of 20%? 

(A) 4 : 1 

(B) 8 : 5 

(C) 7 : 5 

(D) 9 : 5 

Answer: (B) 

 

Q. 67 Asum of ₹ 10000 is lent partly at 8% and remaining at 10% per annum.If the yearly interest on the average is 9.2%, then both parts are respectively. 

(A) ₹5500 and ₹ 4500 

(B) ₹4500 and ₹ 5500 

(C) ₹5000 and ₹ 5000 

(D) ₹4000 and ₹ 6000 

Answer: (D) 

 

Q. 68 The marked price of a bike is ₹ 33,600. The retailer announced a discount of 16% on the occasion of Holi. What amount a customer will have to pay for the bike? 

(A) ₹28224 

(B) ₹5376 

(C) ₹5976 

(D) ₹20224 

Answer: (A) 

Explanation: 

Given, Marked price of the bike = Rs.33600 

Discount = 16% 

Therefore, Selling Price of the bike = 84% of Rs.33600 = Rs.28224. 

 

Q. 69 What is the L.C.M. of 10/21, 20/63, 55/56? 

(A) 5/7

(B) 220/7

(C) 220/504

(D) 5/504

Answer: (B) 

 

Q. 70 The speed of train in going from Bhopal to Varanasi is 100 kmph while when coming back from Varanasi to Bhopal, its speed is 80 kmph. What is the average speed during the whole journey? (correct to two decimal places) 

(A) 78.56 kmph 

(B) 88.89 kmph 

(C) 86.53 kmph 

(D) 90.52 kmph 

Answer: (B) 

 

Q. 71 By selling 20 metres of cloth, a man gains the selling price of 4 metres of cloth. The gain percent is: 

(A) 15 % 

(B) 20 % 

(C) 22 % 

(D) 25 % 

Answer: (D) 

 

Q. 72 A person purchases a silk cloth of length 50 m and width 1 m at the rate of ₹ 250 per sq m and a cotton cloth of length 40 m and width 2m at the rate of ₹ 200 per sq m. What is the difference between the prices of two cloths? 

(A) ₹4100 

(B) ₹3200 

(C) ₹3950 

(D) ₹3500 

Answer: (D) 

Explanation: 

Length of silk cloth = 50 m 

Width of silk cloth = 1 m 

Area of silk cloth = 50*1 = 50 sq.m. 

Price per sq.m = Rs.250 

Total price = Rs.250*50 = Rs.12500 

Length of cotton cloth = 40 m 

Width of cotton cloth = 2 m 

Area of cotton cloth = 40*2 = 80 sq.m. 

Price per sq.m = Rs.200 

Total price = Rs.200*80 = Rs.16000 

Therefore, Difference between the prices = Rs.16000 – Rs.12500 = Rs.3500. 

 

Q. 73 A dice is rolled two times. Find the probability of getting a composite number on first roll and a prime number on second roll? 

(A) ½  

(B) ⅙  

(C) 1/9  

(D) ¼ 

Answer: (B) 

 

Q. 74 A pipe can fill a cistern in 20 minutes whereas the cistern when full can be emptied by a leak in 28 minutes. When both are opened,the time taken to fill the cistern is: 

(A) 48 min 

(B) 70 min 

(C) 80 min 

(D) 60 min 

Answer: (B) 

Explanation: 

Let the capacity of the cistern be 140 units. 

Efficiency of pipe = 140/20 = 7 units per minute 

Efficiency of leakage = 140/-28 = -5 units per minute 

Then, When both are opened, the cistern can be filled at 2 units per minute. 

Therefore, 140 units can be filled in 140/2 = 70 minutes. 

 

Q. 75 Ram buys articles at 70 for ₹ 890 and sold at 60 for ₹ 890. What is his gain percent? 

(A) 17⅓ % 

(B) 25 % 

(C) 20 % 

(D) 16⅔ % 

Answer: (D) 

English 

Instructions 

For the following questions answer them individually 

Q. 76 Select the wrongly spelt word. 

(A) Committee 

(B) Corruption 

(C) Concious 

(D) Completely 

Answer: (C) 

 

Q. 77 Select the meaning of the given idiom. 

Bring to one’s knees 

(A) to force to submit 

(B) to trouble someone 

(C) to request to bend 

(D) to support someone 

Answer: (A) 

 

Q. 78 Select the word which means the same as the group of words given. 

A person whois recovering after an illness or medical treatment 

(A) Athlete 

(B) Convalescent 

(C) Healthy 

(D) Altruist 

Answer: (B) 

 

Q. 79 Select the antonym of the given word. 

RELUCTANT 

(A) Unenthusiastic 

(B) Opposed 

(C) Unwilling 

(D) Willing 

Answer: (D) 

 

Q. 80 Select the most appropriate option to substitute the underlined segment in the given sentence. If there is no need to substitute it, select No substitution required. 

The two sisters look quite different in each other. 

(A) different as each other 

(B) No substitution required 

(C) different with one another 

(D) different from each other 

Answer: (D) 

 

Q. 81 Select the most appropriate option to substitute the underlined segment in the given sentence.If there is no need to substitute it, select No substitution required. 

Our meals usually have at least one item make of some kinds of grain. 

(A) No substitution required 

(B) making of any kinds 

(C) made of some kind 

(D) makes of kind 

Answer: (C) 

 

Q. 82 Select the meaning of the given idiom. 

Cool as Cucumber 

(A) to be smart 

(B) to be aggressive 

(C) to be calm 

(D) to be clever 

Answer: (C) 

 

Q. 83 Select the synonym of the given word. 

MADNESS 

(A) Wisdom 

(B) Stability 

(C) Insanity 

(D) Balance 

Answer: (C) 

 

Q. 84 Select the antonym of the given word. 

LAMENT 

(A) Regret 

(B) Rejoice 

(C) Grieve 

(D) Moum 

Answer: (B) 

 

Q. 85 Identify the segment in the sentence which contains the grammatical error from the given options. 

The thin wire that giving off light is called the filament. 

(A) light is called 

(B) The thin wire 

(C) giving off 

(D) the filament 

Answer: (C) 

 

Q. 86 Identify the segment in the sentence which contains the grammatical error from the given options. 

The jute plant is normal harvested when it is at flowering stage. 

(A) The jute plant 

(B) when it is 

(C) flowering 

(D) is normal 

Answer: (D) 

Instructions 

In the following passage some words have been deleted. Fill in the blanks with the help of the alternatives given. Select the most appropriate option for each number. 

Passage: 

Those of us wholive in (1)…………. covered with forests and surrounded (2)………… hills may findit difficult to (3)…………. what a desert is really like. The (4)…………. belief is that it is (5)………….. endless stretch of sand where no rain falls and, therefore. no vegetation grows. 

Q. 87 Select the most appropriate option that will fill in the blank number 1. 

(A) quarters 

(B) territory 

(C) regions 

(D) cities 

Answer: (D) 

 

Q. 88 Select the most appropriate option that will fill in the blank number 2. 

(A) by 

(B) for 

(C) from 

(D) with 

Answer: (A) 

 

Q. 89 Select the most appropriate option that will fill in the blank number 3. 

(A) feel 

(B) imagine 

(C) look 

(D) calculate 

Answer: (B) 

 

Q. 90 Select the most appropriate option that will fill in the blank number 4. 

(A) popular 

(B) fake 

(C) attractive 

(D) noted 

Answer: (A) 

 

Q. 91 Select the most appropriate option that will fill in the blank number 5. 

(A) the 

(B)

(C) no article needed 

(D) an 

Answer: (D) 

Instructions 

For the following questions answer them individually 

 

Q. 92 Identify the segment in the sentence which contains the grammatical error from the given options. Seeds of mustard plants give us oil and the leave are used as a vegetable. 

(A) Seeds of 

(B) give us oil 

(C) leave are used 

(D) as a vegetable 

Answer: (C) 

 

Q. 93 Select the word which means the same as the group of words given. 

Someone who wishes to destroy the existing government and laws 

(A) Anarchist 

(B) Pacifist 

(C) Patriot 

(D) Hermit 

Answer: (A) 

 

Q. 94 Select the most appropriate option to substitute the underlined segment in the given sentence.If there is no need to substitute it, select No substitution required. 

Tea leaves is separate from the liquid with a strainer, while preparing tea. 

(A) are separated from 

(B) No substitution required 

(C) was separate to 

(D) were separating to 

Answer: (A) 

 

Q. 95 Select the most appropriate word to fill in the blank. 

Among the ………… kinds, the most commonantis black or red. 

(A) separate 

(B) much 

(C) lots of 

(D) various 

Answer: (D) 

 

Q. 96 Select the most appropriate word to fill in the blank. 

I was astounded at his ………. knowledge about the continent of Africa. 

(A) Colossal 

(B) Grave 

(C) Massive 

(D) Vast 

Answer: (D) 

 

Q. 97 Select the most appropriate word to fill in the blank. 

The ……….. of the minister’s statement cannot be verified by people who have no access to official records. 

(A) validity 

(B) rejection 

(C) approval 

(D) ambiguity 

Answer: (A) 

 

Q. 98 Select the most appropriate word to fill in the blank. 

This seems to be a ……… rifle. 

(A) soldier’ 

(B) soldiers 

(C) soldier 

(D) soldier’s 

Answer: (D) 

 

Q. 99 Select the synonym of the given word. 

LUCID

(A) Ambiguous 

(B) Confusing 

(C) Clear 

(D) Unclear 

Answer: (C) 

 

Q. 100 Select the wrongly spelt word. 

(A) Secretary 

(B) Scenary 

(C) Surely 

(D) Sergeant 

Answer: (B) 

SSC GD 5 March 2019 Shift-II Previous Year Paper

SSC GD 5th March 2019 Shift-2 

Reasoning 

Instructions 

For the following questions answer them individually 

Q. 1 Four words have been given out of which three are alike in some manner, while one is different. Choose the odd one. 

(A) Cargo 

(B) Trolley 

(C) Tempo 

(D) Truck 

Answer: A 

Explanation: 

Cargo means goods but other 3 are transport medium, so Cargo is odd one out. 

Option A is correct. 

 

Q. 2 Which of the following answer figure will complete the given figure? 

Answer: C 

Explanation: 

Option C is correct. 

 

Q. 3 If ‘ q ‘ stands for ‘ + ‘ ,’ y ‘ stands for ‘ − ‘,’ z ‘ stands for ‘ ÷ ‘ and ‘ p ‘ stands for ‘ב, then the value of: 

(A) 41 

(B) 31 

(C) 35 

(D) 45 

Answer: A 

 

Q. 4 Dates of birth of some persons are given below. Find out the date of birth of the oldest person: 

A. 12.08.1989 

B. 13.09.1991 

C. 19.06.1991 

D. 20.02.1989 

E. 22.03.1991 

F. 20.01.1991 

G. 20.12.1989 

(A) B 

(B) C 

(C)

(D) D

Answer: D

Explanation: 

Looking at the DOB in the format DD-MM-YYYY we see that D is the oldest. 

Option D is correct. 

 

Q. 5 The statements below are followed by four conclusions labeled I, II, III and IV. Assuming that the information in the statements is true, even if it appears to be at variance with generally established facts, decide which conclusion(s) logically and definitely follow(s) from the information given in the statements. 

Statements: 

Some Linear Equations are Quadratic Equations. 

All Quadratic Equations are Algebraic Equations. 

Conclusions: 

I. All Linear Equations are Algebraic Equations. 

II. Some Algebraic Equations are Linear Equations. 

III. Some Algebraic Equations are Quadratic Equations. 

IV. All Quadratic Equations are Linear Equations. 

(A) Only I and II follows. 

(B) Only II follows. 

(C) Only I follows. 

(D) Only II and III follow. 

Answer: D

Explanation: 

I. All Linear Equations are Algebraic Equations. – False 

II. Some Algebraic Equations are Linear Equations. – True 

III. Some Algebraic Equations are Quadratic Equations. – True 

IV. All Quadratic Equations are Linear Equations. – False 

Option Dis correct. 

 

Q. 6 Select the option that is related to the third term in the same way as the second term is related to the first term.

Bees : Hum :: Snakes : ? 

(A) Hiss 

(B) Hoot 

(C) Roar 

(D) Talk 

Answer: A 

Explanation: 

The pair of words denotes the sounds made, 

Bees hums and snakes hiss. 

Bees : Hum :: Snakes : Hiss 

Option A is correct. 

 

Q. 7 If UPSIDE is coded as 74, then SHARKS will be coded as: 

(A) 68 

(B) 72 

(C) 77 

(D) 76 

Answer: D

Explanation: 

Adding the corresponding values in alphabetical series 

U+P+S+I+D+E = 21+16+19+9+4+5 = 74 

So, S+H+A+R+K+S = 76 

Option Dis correct. 

 

Q. 8 Select the correct option that will fill in the blank and complete the series. 

Mhz, Ngy, Ofx, Pew, ………… 

(A) Qrm 

(B) Rrn 

(C) Qdv 

(D) Qqn 

Answer: C 

Explanation: 

1st Alphabets are +1 (M, N, O, P, Q) 

2nd alphabets are -1 (h, g, f, e, d) 

3rd alphabets are -1 (z, y, x, w, v) 

Option C is correct. 

 

Q. 9 Select the option that is related to the third term in the same way as the second term is related to the first term. Lion : Roar :: Elephant 😕 

(A) Bark 

(B) Bray 

(C) Trumpet 

(D) Grunt 

Answer: C 

Explanation: 

Pair of words denotes the sound of animals. 

Lions roar and an elephant trumpets 

Lion : Roar :: Elephant : trumpets 

Option C is correct. 

 

Q. 10 Find the missing number from the below options. 

(A) 20 

(B) 18 

(C) 36 

(D) 24 

Answer: B 

Explanation: 

(72+18)-(24+17)=49 

(42+36)-(29+31)=18 

Option B is correct 

 

Q. 11 Which of the following answer figure will complete the given figure? 

Answer: D

Explanation: 

Option Dis correct. 

Logic needed 

 

Q. 12 Choose the odd one out from the given alternatives. 

(A) 78 : 4 

(B) 72 : 6 

(C) 39 : 3 

(D) 56 : 4 

Answer: A 

Explanation: 

First number is completely divisible by the second number. 

Option A violates this. i.e 78 ÷ 4 = 19.5 

Option A is correct. 

 

Q. 13 Find the related numbers-pair from the given alternatives. 

12 : 156 :: ? 

(A) 10 : 101 

(B) 15 : 340 

(C) 10 : 110 

(D) 7 : 50 

Answer: C 

Explanation: 

12 × (12 + 1) = 156; 10 × (10 + 1) = 110 

12 : 156 :: 10 : 110 

Option C is correct. 

 

Q. 14 In a queue, Lily is eighteenth from the front while Mini is sixteenth from the back. If Nisha is twenty-fifth from the front andis exactly in the middle of Lily and Mini, then how many persons are there in the queue? 

(A) 47 

(B) 45 

(C) 48 

(D) 46 

Answer: A 

Explanation: 

Lily is 18th and Nisha is 25th from front. Mini is 16th from back 

Number of persons between Lily and Nisha = 6. 

Since Nisha is exactly in the middle of Mini and Lily, so the number of persons between Nisha and Lily = 6. 

So, Number of persons in the queue = (17 + 1 + 6 + 1 + 6 + 1 + 15) = 47. 

Option A is answer. 

 

Q. 15 A piece of paper is folded and cut as shown below in the question figures from the given answer figures, indicate the pattern that it will appear when it is opened ? 

Answer: C 

 

Q. 16 Choose the option figure in which the problem figure is hidden/embedded. 

Answer: C 

 

Q. 17 In a certain code ROHIT is written as “TQIKV”, how will COUNTRY be coded? 

(A) EQWPUTA 

(B) EQWVPTE 

(C) EQWVPAT 

(D) EQWOVTA 

Answer: D

 

Q. 18 Four letter clusters have been given out of which three are alike in some manner, while one is different. Choose the odd one 

(A) CFJ 

(B) GJM 

(C) TWZ 

(D) DGJ 

Answer: A 

Explanation: 

A. C → (+3)F → (+4)J 

B. G → (+3)J → (+3)M 

C. T → (+3)W → (+3)Z 

D. D→ (+3)G → (+3)J 

Option A is correct. 

 

Q. 19 Select the Venn diagram that best illustrates the relationship between the three given classes. Girls, Healthy, Trains 

Answer: A 

 

Q. 20 Select the correct option that will fill in the blank and complete the series. 

1331, 121, 2197, 169, 3375, ………. 

(A) 216 

(B) 215 

(C) 225 

(D) 205 

Answer: C 

Explanation: 

1331 = 113 

121 = 112 

2197 = 133 

169 = 132 

3375 = 153 

225 = 152 

Option C is correct 

 

Q. 21 A, B, C, D, E, F, G and H are sitting around circle facing at the centre. E, is second to the left of F and third to the right of A. B is third to the right of G who is not an immediate neighbour of E or F. C is second to the right of B. Dis to the immediate left of A and third to the left of H. 

Whois the fifth to the right of C? 

(A)

(B)

(C) B 

(D)

Answer: D

 

Q. 22 Select the correct option that will fill in the blank and complete the series. 

37, 26, 39, 24, 41, ………… 

(A) 22 

(B) 26 

(C) 23 

(D) 25 

Answer: A 

Explanation: 

37-26=11 

26-39 =-13 

39-24 = 15 

24 -41 = -17 

41- 22 =19 

37, 26, 39, 24, 41, 22 

Option A is correct. 

 

Q. 23 If a mirror is placed on MN line, then which of the answer figures will be the right image of the given figure? 

Answer: A 

 

Q. 24 The statements below are followed by two conclusions labeled I and II. Assuming that the information in the statements is true, even if it appears to be at variance with generally established facts, decide which conclusion(s) logically and definitely follow(s) from the information given in the statements. 

Statements: 

All rivers are mountains. 

All forests are mountains. 

Conclusion: 

I. Some Forests are river. 

II. Noforestis a river. 

(A) Neither conclusion I nor II follows 

(B) Only conclusion II follows 

(C) Only conclusion I follows 

(D) Either conclusion I or II follows 

Answer: D

 

Q. 25 Select the option that is related to the third term in the same way as the second term is related to the first term. 

pRSq : tVWu :: iKLj : ? 

(A) MrsQ 

(B) MopN 

(C) mRSq 

(D) mOPn 

Answer: D

General knowledge 

Instructions 

For the following questions answer them individually 

Q. 26 Minimum Support Price for 2018-19 for Maize is set at ………… 

(A) ₹2,430 per quintal 

(B) ₹1,950 per quintal 

(C) ₹1,700 per quintal 

(D) ₹5,150 per quintal 

Answer: C 

 

Q. 27 The Charter Act of 1833 made the Governor-General of ………… as the Governor-Generalof India and vested in him all civil and military powers. 

(A) Oudh State 

(B) Bengal 

(C) Berar Province 

(D) United Provinces 

Answer: B 

 

Q. 28 The Maithon Dam is in which state? 

(A) Chhattisgarh 

(B) Jharkhand 

(C) Odisha 

(D) Madhya Pradesh 

Answer: B 

 

Q. 29 Which amendment to the constitution introduced the Goods and Services Tax (GST)? 

(A) 103rd Constitution Amendment Act, 2018 

(B) 73rd Constitution Amendment Act of 2003 

(C) 101st Constitution Amendment Act, 2016 

(D) 92nd Constitution Amendment Act of 2003 

Answer: C 

 

Q. 30 Which Indian was crowned Miss World Continental Queen of Beauty—Asia & Oceania in 1999? 

(A) Dia Mirza 

(B) Yukta Mookhey 

(C) Reita Faria 

(D) Priyanka Chopra 

Answer: B 

 

Q. 31 Which team won in the 2018 Pro Wrestling League finals? 

(A) Punjab Royals 

(B) Delhi Sultans 

(C) Haryana Hammers 

(D) Veer Marathas 

Answer: A 

 

Q. 32 Name the highest peak of the Satpura range. 

(A) Dbupgarh 

(B) Kalsubai 

(C) Doddabetta 

(D) Anamudi 

Answer: A 

 

Q. 33 Moment of inertia of a thin rod of mass ‘M’ and length ‘L’ about the axis perpendicular to the rod and passing through its centre is ………… 

(A) ML2 /4
(B) ML2/6

(C) ML2/12
(D) ML2/2

Answer: C 

 

Q. 34 Name the class of folk songs of Goa, usually sung to mourn the death of the loved ones. 

(A) Sumangali 

(B) Banvarh 

(C) Gha To Kito 

(D) Khubakeshei 

Answer: B 

 

Q. 35 The formula of Cyanogen is ……….. 

(A) CH3OCN 

(B) C2N2 

(C) CH2CHCN 

(D) CH3CN 

Answer: B 

 

Q. 36 The coccyx is part of the ………….. 

(A) spine 

(B) shoulder girdle 

(C) arm 

(D) leg 

Answer: A 

 

Q. 37 Parliamentary form of government as laid down in the Indian Constitution is influenced by the constitution of ………….. 

(A) United Kingdom 

(B) Australia 

(C) United States 

(D) Ireland 

Answer: A 

 

Q. 38 Name the Sanskrit scholar who was awarded the Bharat Ratna? 

(A) Madan Mohan Malaviya 

(B) Pandurang Vaman Kane 

(C) Purushottam Das Tandon 

(D) Bidhan Chandra Roy 

Answer: B 

 

Q. 39 ………….. alloy consists of copper and tin. 

(A) Bell Metal 

(B) Solder 

(C) Rose Metal 

(D) German silver 

Answer: A 

 

Q. 40 Which of the following dances is a dance form from Chhattisgarh? 

(A) Raut Nacha 

(B) Chiraw 

(C) Ottam Thullal 

(D) Macha 

Answer: A 

 

Q. 41 Which of the following is an autobiography by former Minister in charge of External Affairs, Kunwar Natwar Singh? 

(A) Courage and Conviction 

(B) Where Have I Been? 

(C) Ace Against Odds 

(D) One Life is not Enough 

Answer: D

 

Q. 42 The Kila Raipur Rural Olympicsis held annually in between January and February near ……… 

(A) Gwalior 

(B) Ludhiana 

(C) Vadodara 

(D) Aizwal 

Answer: B 

 

Q. 43 It was stated in the Union Budget 2018, that the Ujjwala Yojana, the free LPG connection scheme would be expanded to …………. crore Women. 

(A) eight 

(B) six 

(C) two 

(D) four 

Answer: A 

 

Q. 44 Chandragupta I was succeeded by his son …………. 

(A) Samudragupta 

(B) Bhanugupta 

(C) Skandagupta 

(D) Purugupta 

Answer: A 

 

Q. 45 The origin of the sun was approximately how many years before the present age? 

(A) 4.6 billion years 

(B) 10.6 billion years 

(C) 1.5 billion years 

(D) 2.4 billion years 

Answer: A 

 

Q. 46 India defeated which team to win the Nidahas Trophy 2018 cricket Tri-Series Final played in March 2018? 

(A) South Africa 

(B) West Indies 

(C) Bangladesh 

(D) England 

Answer: C 

 

Q. 47 Which British retired Civil Service Officer took the initiative to convene the first meeting of the Indian National Union? 

(A) John Morley 

(B) Edwin Montagu 

(C) Allan Octavian Hume 

(D) Gilbert Elliot 

Answer: C 

 

Q. 48 The Economic Survey report 2017-18 stated that the Services sector growth for FY 18 was likely to be at ……….. 

(A) 8.30% 

(B) 6.50% 

(C) 9.60% 

(D) 7.10% 

Answer: A 

 

Q. 49 Farad is the unit of ……….. 

(A) electric conductance 

(B) capacitance 

(C) magnetic flux 

(D) magnetic flux density 

Answer: B 

 

Q. 50 In Battle of Khatoli (1517) Rana Sanga of Mewar defeated Sultan of Delhi, ………… Lodhi and imprisoned him and then released him after collecting ransom. 

(A) Bahlul Khan 

(B) Qutb-ud-din 

(C) Ibrahim 

(D) Sikandar 

Answer: C 

Quant 

Instructions 

For the following questions answer them individually 

Q. 51 The mode of the data 26, 32, 26, 28, 26, 24, 31, 24 is: 

(A) 28 

(B) 24 

(C) 31 

(D) 26 

Answer: D

Explanation: 

MODE : The mode of a set of data values is the value that appears most often. 

The mode of the data 26, 32, 26, 28, 26, 24, 31, 24 is 26 because 26 has appeared the most number of times. Option Dis correct. 

 

Q. 52 What should come in place of the question mark (?) in the following question? 

[((16 ÷ 4) × 4) ÷ 4] =? 

(A)

(B)

(C)

(D)

Answer: C 

Explanation: 

Using BODMAS rule, 

[((16 ÷ 4) × 4) ÷ 4] 

[(4 × 4) ÷ 4] = 4 

Option C is correct. 

 

Q. 53 Ajay can row hisboatin still water, at a speed of 6 kmph.If the speed ofthe stream is 4 kmph, how long will he take to a distance of 30 km downstream? 

(A) 2 hours 

(B) 5 hours 

(C) 4 hours 

(D) 3 hours 

Answer: D

 

Q. 54 An inlet pipe takes 8 hours to fill a tank. An outlet pipe takes 12 hours to empty it. If both pipes are opened simultaneously, in how many hours will the tank be filled? 

(A) 36 

(B)

(C) 24 

(D) 12 

Answer: C 

 

Q. 55 If the profit made by a merchant onselling 45 m of cloth is equal to the selling price of 5m of cloth, then his profit percentage is: 

(A) 9.09% 

(B) 12.5% 

(C) 11.11% 

(D) 10% 

Answer: B 

 

Q. 56 Two parallelogram stand on equal bases and between the same parallel. The ratio of their areas is: 

(A) 1 : 1 

(B) 2 : 1 

(C) 1 : 3 

(D) 1 : 2 

Answer: A 

 

Q. 57 Rajaram purchased a certain amount of wheat. If he sold one-fourth of the wheat at a profit of 12%, then the profit percent he needs to make on the remaining wheat to make an overall profit of 15% is: 

(A) 18% 

(B) 16% 

(C) 15% 

(D) 20% 

Answer: B 

 

Q. 58 Which of the following is the greatest among all? 

(A) 8% discount on marked price ₹400 

(B) 12% discount on marked price ₹240 

(C) 7% discount on marked price ₹500 

(D) 10% discount on marked price ₹320 

Answer: C 

 

Q. 59 Sanjay scored 75 out of 150 marks in his mid-term exam and 105 out of 150 in his annual exam. The percentage increase in his marks is: 

(A) 30% 

(B) 45% 

(C) 50% 

(D) 40% 

Answer: D

 

Q. 60 The average marks of 42 students in an examination, was 64. Later it was found that the marks of three students were misread as 72, 58 and 44 instead of the actual marks 64, 42 and 26 respectively. What is the correct average marks? 

(A) 61 

(B) 62 

(C) 63 

(D) 60 

Answer: C 

 

Q. 61 If the area of the base of a cone is increased then it becomes 1.96 times of original area. Its volume is increased by: 

(A) 141% 

(B) 40% 

(C) 96% 

(D) 100% 

Answer: C 

 

Q. 62 The pie chart given here shows the spendings of a country on various sports during a particular year. Study the graph carefully and answer the questions given below it. 

If the total amount spent on sports during the year be ₹18000, the amount spent on Cricket exceeds that on football by (in ₹): 

(A) 2800 

(B) 1350 

(C) 3000 

(D) 2550 

Answer: B 

 

Q. 63 Five years ago, the ratio of the ages of a father and his son was 5 : 3. Which of the following cannot be the ratio of their ages 10 years from now? 

(A) 3:2 

(B) 8:7 

(C) 6:5 

(D) 7:3 

Answer: D

 

Q. 64 In what time does money becomes double at simple interest rate of 12% per annum? 

(A) 6⅓ Years

(B) 6⅔ Years 

(C) 8⅓ Years 

(D) 8⅔ Years 

Answer: C 

 

Q. 65 If 30 men can do a job in 30 days, find what part of the job can be completed by 1 man in 1 day. 

(A) 1/30

(B) 1/900

(C) 1/90

(D) 1/60

Answer: B 

 

Q. 66 The ratio of length of two trains is 6 : 5 and theratio of their speed is 3 : 2 The ratio of time taken by them to cross a pole is: 

(A) 3 : 5 

(B) 4 : 5 

(C) 5 : 6 

(D) 5 : 8 

Answer: B 

 

Q. 67 The pie chart given here shows the spendings of a country on various sports during a particular year. Study the chart carefully and answer the questions given below it. 

How much percent less is spent on Football than that on Cricket? 

Answer: A 

 

Q. 68 A certain sum is divided among p, q and r in a manner that for every rupee that p gets, q gets 75 paise and for every rupee that q gets, r gets 50 paise. If r’s share in the total sum is ₹ 36, then find the share of p. 

(A) ₹96 

(B) ₹72 

(C) ₹54 

(D) ₹60 

Answer: A 

 

Q. 69 What should come in place of the question mark (?) in the following question? 

6 ÷ 6 × 9 + 6 − 9 × 6 − 6 + 6 × 9 =? 

(A)

(B)

(C) 10 

(D)

Answer: D

Explanation: 

6 ÷ 6 × 9 + 6 − 9 × 6 − 6 + 6 × 9 =? 

Using BODMAS rule, 

= 1 × 9 + 6 − 9 × 6 − 6 + 6 × 9 

= 9 + 6 − 54 − 6 + 54 = 9 

Option Dis correct. 

 

Q. 70 Find the average of (5 + 5 + ………. upto 200 times) and (8 + 8 + ………… upto 100 times). 

(A) 6.5 

(B)

(C)

(D) 75 

Answer: C 

 

Q. 71 If the HCF of two numbers is 6 and their LCM is 120, one such pair of numbers is: 

(A) 24, 30 

(B) 18, 40 

(C) 12, 60 

(D) 12, 40 

Answer: A 

Explanation: 

Product of two numbers = 120 × 6 = 720 

LCM × HCF 

24 × 30 = 720 = LCM × HCF 

Option A is correct. 

 

Q. 72 In class there are a total of 60 boys and girls. Which of the following can’t represent the ratio of the number of boys and girls in the class? 

(A) 2 : 3 

(B) 1 : 3 

(C) 1 : 6 

(D) 3 : 7 

Answer: C 

Explanation: 

In class there are a total of 60 boys and girls. 

Let x = 60, Now checking all the options, 

(A) 2 : 3 => 2x+3x = 5x = 60 =>x =12 

1 : 3 => 1x+3x = 4x = 60 =>x =15 

1 : 6 => 1x+6x = 7x = 60 =>x =8.571 (Value is in decimal) 

3 : 7 => 3x+7x = 10x => 60 =>x =6 

Option C is correct. 

 

Q. 73 A sum borrowed under compound interest doubles itself in 10 years. When will it become fourfold of itself at the same rate of interest? 

(A) 24 years 

(B) 15 years 

(C) 20 years 

(D) 40 years 

Answer: C 

 

Q. 74 The pie chart given here shows the spendings of a country on various sports during a particular year. Study the chart carefully and answer the questions given below: 

What percent of the total spendings is spent on others? 

(A) 12.5% 

(B) 45% 

(C) 25% 

(D) 22.5% 

Answer: B 

 

Q. 75 If the surface area of two spheres is in the ratio 49 : 25, then the ratio of their volumes will be: 

(A) 64 : 27 

(B) 25 : 49 

(C) 343 : 64 

(D) 343 : 125 

Answer: D

English 

For the following questions answer them individually 

Q. 76 Select the synonym of the given word. 

Delight. 

(A) Grief 

(B) Joy 

(C) Excitement 

(D) Dread 

Answer: B 

 

Q. 77 Select the word which means the same as the group of words given. 

Existing, happening, or operating in the air 

(A) Terrestrial 

(B) Aerial 

(C) Arboreal 

(D) Aquatic 

Answer: B 

 

Q. 78 Select the synonym of the given word. 

Incredible 

(A) Unbelievable 

(B) Ordinary 

(C) Unspectacular 

(D) Impossible 

Answer: A 

 

Q. 79 Select the most appropriate option to substitute the underlined segment in the given sentence. If there is no need to substitute it, select No substitution required. 

Modern farming practices is involving the use of fertilizers and pesticides. 

(A) involved the using of 

(B) involve the use of 

(C) has involved the use in 

(D) No substitution required 

Answer: B 

 

Q. 80 Select the most appropriate word to fill in the blank. 

The French Revolution …………. to the end of monarchyin France. 

(A) is leading 

(B) were leading 

(C) led 

(D) was led 

Answer: C 

 

Q. 81 Identify the segment in the sentence which contains the grammatical error from the given options. Scholars have studied the lives of present-day farmer’s who practice agriculture. 

(A) farmer’s 

(B) the lives of 

(C) Scholars have 

(D) practice agriculture 

Answer: A 

 

Q. 82 Identify the segment in the sentence which contains the grammatical error from the given options. If you plant a seed, you will be noticed that it takes some time to grow. 

(A) to grow 

(B) takes some 

(C) will be noticed 

(D) If you 

Answer: C 

 

Q. 83 Select the most appropriate word to fill in the blank. 

Minerals are ……….. occurring substances which have certain physical properties. 

(A) thoroughly 

(B) naturally 

(C) fictitiously 

(D) falsely 

Answer: B 

Instructions 

In the following passage some words have been deleted. Fill in the blanks with the help of the alternatives given. Select the most appropriate option for each number. 

Passage: 

On the morning of 14 July 1789, (1)…………. city of Paris was ina state of (2)…………. . The king had (3)…………. troops to moveinto the city. Rumors spread that he would soon order the army to (4)………….. fire upon the citizens. Some 7,000 men and womengathered (5)……….. the town hall. 

 

Q. 84 Select the most appropriate option that will fill in the blank number 1. 

(A) No article needed 

(B) an 

(C) the 

(D)

Answer: C 

 

Q. 85 Select the most appropriate option that will fill in the blank number 2. 

(A) pease 

(B) disagreement 

(C) ease 

(D) alam 

Answer: D

 

Q. 86 Select the most appropriate option that will fill in the blank number 3. 

(A) commanded 

(B) urged 

(C) expected 

(D) announced 

Answer: A 

 

Q. 87 Select the most appropriate option that will fill in the blank number 4. 

(A) open 

(B) cross 

(C) Spread fire 

(D) extinguish 

Answer: A 

 

Q. 88 Select the most appropriate option that will fill in the blank number 5. 

(A) in front of 

(B) besides 

(C) across 

(D) on top of 

Answer: A 

Instructions 

For the following questions answer them individually 

 

Q. 89 Select the wrongly spelt word. 

(A) Bracelet 

(B) Biannual 

(C) Bileive 

(D) Brochure 

Answer: C 

 

Q. 90 Select the wrongly spelt word. 

(A) Auditorum 

(B) Analysis 

(C) Absolute 

(D) Ambition 

Answer: A 

 

Q. 91 Select the most appropriate option to substitute the underlined segment in the given sentence.If there is no need to substitute it, select No substitution required. 

The aquatic animals can breathed under water due to dissolved oxygen. 

(A) can breathe under water 

(B) can be breathing in water 

(C) could have breathed under water 

(D) No substitution required 

Answer: A 

 

Q. 92 Select the most appropriate wordto fill in the blank. 

Most workers …………. as laborers in factories where their wages were fixed. 

(A) is employed 

(B) has employed 

(C) were employed 

(D) was employ 

Answer: C 

 

Q. 93 Select the antonym of the given word. 

Genuine 

(A) Thankful 

(B) Fake 

(C) Unhealthy 

(D) Original 

Answer: B 

 

Q. 94 Select the meaning of the given idiom. 

Fish out of water 

(A) To shift to a new place 

(B) In a new job 

(C) To be uncomfortable 

(D) To be depressed 

Answer: C 

 

Q. 95 Select the most appropriate word to fill in the blank. 

Biosphere is a narrow zone of the earth where land, water and air ………… with each other to support life. 

(A) collapse 

(B) interact 

(C) surround 

(D) impose 

Answer: B 

 

Q. 96 Identify the segment in the sentence which contains the grammatical error from the given options. Raw materials are substances that are either found naturally or produce by farmers. 

(A) found naturally 

(B) produce by 

(C) Raw materials 

(D) substances 

Answer: B 

 

Q. 97 Select the word which means the same as the group of words given. 

That which can be heard 

(A) Legible 

(B) Edible 

(C) Tangible 

(D) Audible 

Answer: D

 

Q. 98 Select the most appropriate option to substitute the underlined segment in the given sentence.If there is no need to substitute it, select No substitution required. 

Tissue is a group of cells that are similar to structure and are organised together. 

(A) that were similarly in 

(B) No substitution required 

(C) that is similarly to 

(D) that are similar in 

Answer: D

 

Q. 99 Select the meaning of the given idiom. 

(A) piece of cake 

(A) Unrealistic hope 

(B) A small quantity 

(C) A very easy task 

(D) Very tasty 

Answer: C 

 

Q. 100 Select the antonym of the given word. 

Rational 

(A) Impatient 

(B) Conscious 

(C) Illogical 

(D) Polite 

Answer: C 

SSC GD 3 March 2019 Shift-III Previous Year Paper

SSC GD 3rd March 2019 Shift-3 

Reasoning 

Instructions 

For the following questions answer them individually 

Q. 1 A piece of paper is folded and cut as shown in the question figures. From the given figures, indicate how it will appear when opened? Figure 

(A) Figure 1 

(B) Figure 2 

(C) Figure 3 

(D) Figure 4 

Answer: D

 

Q. 2 In a class of 45 students, a boy is ranked 20th When two boys joined, his rank was dropped by one. What is his new rank from the end? 

(A) 27th 

(B) 26th 

(C) 25th 

(D) 28th 

Answer: A

Explanation: 

In a class of 45 students, a boy is ranked 20th When two boys joined, 

The rank of the boy dropped by 1, i.e New rank is 21st 

Total students =45+2 =47 

His new rank from the end: 

47 – 21 + 1 

47 – 20 =27 th 

Option Ais correct. 

 

Q. 3 Which of the following answer figure will complete the given figure? 

Answer: C

 

Q. 4 Study the following Venn-diagram and answer the given question. 

Which letter represents a male politician only? 

(A)

(B)

(C)

(D)

Answer: D

 

Q. 5 The statements below are followed by two conclusions labeled I and II. Assuming That the information in the statements is true, even ifit appears to be at variance with generally established facts, decide which conclusion(s) logically and definitely follow(s) from the information given in the statements. 

Statements: 

Some scooters are planes. 

Some planes are dogs. 

Some dogs are plants. 

Conclusions: 

I. Someplants are scooters. 

II. No plane is scooter. 

(A) Only Conclusion I follows. 

(B) Neither Conclusion I nor II follows. 

(C) Only Conclusion II follows. 

(D) Either Conclusion I or II follows. 

Answer: B

Explanation: 

Conclusions: 

I. Some plants are scooters – False 

II. No plane is scooter – False 

Neither I nor II follows 

Option Bis correct. 

 

Q. 6 Select the option that is related to the third term in the same way as the first term is related to the second term. 

AFK : EJO :: PUZ : ? 

(A) TXE 

(B) TY(D) 

(C) UOM 

(D) US(C) 

Answer: B

Explanation: 

(A) → (+5)F → (+5)K : E → (+5)J → (+5)O : 

: P → (+5)U → (+5)Z : T → (+5)Y → (+5)(D) 

Option Bis correct. 

 

Q. 7 The statements below are followed by two conclusions labeled I and II. Assuming That the information in the statements is true, even if it appears to be at variance with generally established facts, decide which conclusion(s) logically and definitely follow(s) from the information given in the statements. 

Statements: 

All kids are Adults. 

Some Adults are old. 

Conclusion: 

I. All old are kids. 

II. Some kids are old. 

(A) Only conclusion I follows 

(B) Either conclusion I or II follows 

(C) Only conclusion II follows 

(D) Neither conclusion I nor II follows 

Answer: D

Explanation: 

I. All old are kids – False 

II. Some kids are old – False 

Neither I nor II follows 

Option Dis correct. 

 

Q. 8 Four words have been given out of which three are alike in some manner, while one is different. Choose the odd one. 

(A) Kathakali 

(B) Kuchipudi 

(C) Sattriya 

(D) Hemis 

Answer: D

Explanation: 

Kathakali, Kuchipudi and Sattriya are dance forms but Hemis is a National Park 

So Hemis is odd one out 

Option Dis correct 

 

Q. 9 In a certain code, ANIMAL is coded as INALAM, then how is SAMPLE coded in the same way? 

(A) MALESP 

(B) MSALEP 

(C) ASMALE 

(D) MASELP 

Answer: D

Explanation: 

Option Dis correct. 

 

Q. 10 Select the option that is related to the third term in the same wayas the first term is related to the second term. 

35 : 15 :: 72 : ? 

(A) 54 

(B) 27 

(C) 15 

(D) 14 

Answer: D

Explanation: 

3 × 5 = 

35: 15 

72: 

7 × 2 = 14 

35 : 15 :: 72 : 14 

Option Dis correct. 

 

Q. 11 If SENSIBLE is written as 10 and SAMUELS as 9, how will NADAL be written? 

(A)

(B)

(C) 11 

(D)

Answer: D

Explanation: 

SENSIBLE = Total no. of letters +2 = 8+2=10 

SAMUELS = 7+2= 9 

NADAL = 5+2 =7 

Option Dis correct. 

 

Q. 12 Choose the option figure in which the problem figure is hidden/embedded. 

Answer: D

 

Q. 13 Select the correct option that will fill in the blank and complete the series. 

WVU, RQP, MLK, ? 

(A) HGF 

(B) JHI 

(C) HGI 

(D) IGF 

Answer: A

Explanation: 

If we look at the given series, 

WVU -> these 3 letters are in series and in reverse order 

U – 3 = R 

RQP -> these 3 letters are in series and in reverse order 

P – 3 = M 

MLK -> these 3 letters are in series and in reverse order 

K – 3 = H 

So, the next term will be HGF. 

 

Q. 14 Four options have been given out of which three are alike in some manner, while one is different. Choose the odd one. 

(A) 693 

(B) 385 

(C) 287 

(D) 473 

Answer: C

Explanation: 

693 = 6 + 9 + 3 = 18 (even) 

385 = 3 + 8 + 5 = 16 (even) 

287 = 2 + 8 + 7 = 17 (odd) 

473 = 4 + 7 + 3 = 14 (even) 

Option Cis correct. 

 

Q. 15 Identify the two figures from the following set of figures which are out of position and require interchange of positions to put the entire series in order. 

(A) (B & D) 

(B) (D & E) 

(C) (D & F) 

(D) (A & C) 

Answer: B

 

Q. 16 Five monkeys A, B, C, Dand E are sitting on a branch of a tree. 

C is sitting next to D. Dis not sitting with E.E is on the left end of the branch. Cis on second position from right. Ais immediate right of Band Bis on the right side of E. Who is sitting at the centre? 

(A) D

(B) C

(C) A

(D) B

Answer: C

 

Q. 17 Select the correct combination of mathematical signs to replace ‘*’ signs and to balance the given equation: 

78 * 3 * 4 * 36 * 140 

(A) − + × =

(B) − + × =

(C) − + × =

(D) − + × =

Answer: C

 

Q. 18 Select the correct option that will fill in the blank and complete the series. 

145, 100, 65, 40, ? 

(A) 15 

(B) 10 

(C) 20 

(D) 25 

Answer: D

Explanation: 

145, 100, 65, 40, ? 

145-100 = 45 

100-65 = 35 

65-40 = 25 

40-15 = 25 

So, there is a difference of 15,25,35,45 between each numbers. 

Option Dis correct. 

 

Q. 19 Find the missing number from the below options. 

(A)

(B)

(C)

(D)

Answer: B

 

Q. 20 X, Y, Z, P, Q and R are sitting in a row facing north. X is the neighbour of Y and P. Q is the neighbour of Z and R, P is to the right of X and P and Z are immediate neighbours. 

Which pair sits on the extreme ends? 

(A) Y, P 

(B) Y, R 

(C) X, P 

(D) P, Z 

Answer: B

 

Q. 21 If a mirror is placed on the line MN, then which of the answer figures is the right image of the given figure? 

Answer: B

 

Q. 22 Select the correct option that will fill in the blank and complete the series. 

2, 8, 32, 128, 512, ? 

(A) 2507 

(B) 2536 

(C) 2067 

(D) 2048 

Answer: D

Explanation: 

2, 8, 32, 128, 512, 2048 

21 = 2 

23 = 8 

25 = 32 

27 = 128 

29 = 512 

211 = 2048 

Option Dis correct. 

 

Q. 23 Four letter clusters have been given out of which three are alike in some manner, while one is different. Choose the odd one. 

(A) JOT 

(B) HMR 

(C) INS 

(D) DLT 

Answer: D

 

Q. 24 Select the related word pair from the given alternatives. 

India : Rupee :: …….. : ……… 

(A) Switzerland : Euro 

(B) Russia: Ruble 

(C) South Africa : Pound 

(D) China : Yen 

Answer: B

Explanation: 

India : Rupee :: Russia: Ruble 

Country : Currency 

Option Bis correct. 

 

Q. 25 Select the option that is related to the third term in the same way as the first term is related to the second term. 

Hand : Finger :: Leg : ……… 

(A) Heel 

(B) Toes 

(C) Walk 

(D) Knee 

Answer: B

Explanation: 

Hand : Finger :: Leg : Toe 

We have finger in our hand and have toe in our leg. 

Option Bis correct. 

General knowledge 

Instructions 

For the following questions answer them individually 

Q. 26 The women’s team from which country won the 2018 Carrom World Cup, held in South Korea? 

(A) South Korea 

(B) Japan 

(C) India 

(D) China 

Answer: C

 

Q. 27 Who won the 2018 French Open Men’s Tennis? 

(A) Roger Federer 

(B) Rafael Nadal 

(C) Novak Djokovic 

(D) Andy Murray 

Answer: B

 

Q. 28 The humerus is part of the ………. 

(A) spine 

(B) shoulder girdle 

(C) leg 

(D) arm 

Answer: D

 

Q. 29 The ……… was an Actof the Parliamentof Great Britain intended to overhaul the managementofthe East India Company’s rule in India. 

(A) Pitt’s India Act 

(B) Government of India Act of 1858 

(C) Indian Councils Act of 1861 

(D) Regulating Act 1773 

Answer: D

 

Q. 30 The fundamental rights of citizens are embodied in which part of the Indian Constitution? 

(A) Part I 

(B) Part IV(A) 

(C) Part VII 

(D) Part III 

Answer: D

 

Q. 31 The harvest festival, Puthari is celebrated by which tribes? 

(A) Angami 

(B) Garo 

(C) Kodava 

(D) Monpa 

Answer: C

 

Q. 32 Which of the following is a book by Vikram Seth? 

(A) The Ocean of Churn 

(B) River of Smoke 

(C) Riot At Misri Mandi 

(D) The Mistress of Spices 

Answer: C

 

Q. 33 Name the Indian ruler of Harkanya dynasty who was the son of Ajatashatru and who laid the foundation of the city of Pataliputra. 

(A) Pradyota 

(B) Udayin 

(C) Mahanandin 

(D) Nandivardhana 

Answer: B

 

Q. 34 Minimum Support Price for 2018-19 for Jowar of the Hybrid variety is set at …….. 

(A) ₹1,950 per quintal 

(B) ₹1,750 per quintal 

(C) ₹2,430 per quintal 

(D) ₹5,150 per quintal 

Answer: C

 

Q. 35 Centripetal force ‘F’ required for circular motion of an object of mass ‘m’ movingin circle of radius ‘r’ at a tangential velocity ‘v’ is equal to ………. 

(A)  mr/v2
(B)  mv2/r

(C)  mv/r2
(D)  mr2 / v

Answer: B

 

Q. 36 Which city is NOT situated on the Banks of the Chambal River? 

(A) Jabalpur 

(B) Kota 

(C) Gwalior 

(D) Dholpur 

Answer: A

 

Q. 37 Fundamental duties in the Indian constitution are borrowed from the constitution of …………. 

(A) France 

(B) Canada 

(C) USSR 

(D) Britain 

Answer: C

 

Q. 38 Titan the second largest moon in our solar system orbits around the planet …….. 

(A) Uranus 

(B) Jupiter 

(C) Neptune 

(D) Saturn 

Answer: D

 

Q. 39 The Union Budget 2018 announced that Operation Green will be launched for agriculture with a corpus of ………… 

(A) ₹1,000 crore 

(B) ₹500 crore 

(C) ₹5,000 crore 

(D) ₹100 crore 

Answer: B

 

Q. 40 Name the unit in which the calorific value of fuel is expressed. 

(A) Candela 

(B) Kilojoule 

(C) Kelvin 

(D) Kilogram 

Answer: B

 

Q. 41 Which gland controls the functioning of other endocrine glands? 

(A) Thyroid Gland 

(B) Pineal Gland 

(C) Adrenal glands 

(D) Pituitary gland 

Answer: D

 

Q. 42 Name the first Indian to qualify for the Indian Civil Service. 

(A) V. O. Chidambaram Pillai 

(B) Satyendranath Tagore 

(C) Bankim Chandra Chatterjee 

(D) Dadabhai Naoroji 

Answer: B

 

Q. 43 Which of the following dances is a dance form from Arunachal Pradesh? 

(A) Popir 

(B) Chiraw 

(C) Lezim 

(D) Macha 

Answer: A

 

Q. 44 The battle of Karnal (1739) was won by Persian King Nadir Shah by defeating the army of ………. 

(A) Mughal Emperor, Muhammad Shah 

(B) Shuja-ud-Daulah, the Nawab of Awadh 

(C) Ghulam Muhammad Ghouse Khan 

(D) Ghiyas-ud-din Balban 

Answer: A

 

Q. 45 Name the Bharat Ratna recipient who was a social reformer and educator. He played a vital role in the upliftment of women. 

(A) Bidhan Chandra Roy 

(B) M Visvesvaraya 

(C) Govind Ballabh Pant 

(D) Dhondo Keshav Karve 

Answer: D

 

Q. 46 Name the devotional folk music of Manipur usually sung by a group of people. 

(A) Domkach 

(B) Khubak eshei 

(C) Gha To Kito 

(D) Banvarh 

Answer: B

 

Q. 47 The Economic Survey report 2017-18 stated that the Agriculture growth in FY18 was likely to be at ………. 

(A) 4.30% 

(B) 2.10% 

(C) 3.90% 

(D) 5.50% 

Answer: B

 

Q. 48 Which island emerged in the Bay of Bengal in the after math of the Bhola cyclone in 1970? 

(A) Lohachara Island 

(B) Jambudwip 

(C) Ghoramara Island 

(D) New Moore Island 

Answer: D

 

Q. 49 Tesla is the unit of …………. 

(A) electric conductance 

(B) magnetic flux density 

(C) capacitance 

(D) magnetic flux 

Answer: B

 

Q. 50 Whois the first Indian to win the Miss Earth pageant since its inception in 2001? 

(A) Dia Mirza 

(B) Reita Faria 

(C) Nicole Faria 

(D) Priyanka Chopra 

Answer: C

Quant 

Instructions 

For the following questions answer them individually 

Q. 51 Study the bar chart and answer the question based on it. 

What was the percentage increase in the production of fertilizers in 2001 compared to that in 2000? 

(A) 40% 

(B) 30% 

(C) 25% 

(D) 50% 

Answer: D

 

Q. 52 If 15 workers can earn ₹1,800 in 10 days. Find the earning (in ₹) of 5 workers in 8 days. 

(A) ₹ 540 

(B) ₹ 480 

(C) ₹ 360 

(D) ₹ 400 

Answer: B

 

Q. 53 Simplify: 7.8 – 0.4 of (5.1 – 3.8) + 9.3 x 1.5 

(A) 23.12 

(B) 12.23 

(C) 21.23 

(D) 23.21 

Answer: C

Explanation: 

7.8 − 0.4of(5.1 − 3.8) + 9.3 × 1.5 

Using BODMAS, 

= 7.8 − 0.4of(5.1 − 3.8) + 13.95 

= 7.8 − 0.4of1.3 + 13.95 

= 7.8 − 0.4 × 1.3 + 13.95 

= 7.8 − 0.52 + 13.95 

=21.75 – 0.52 

= 21.23 

Option Cis correct. 

 

Q. 54 A is twice as good as a workman as B. And together, they finish a piece of work in 20 days. In how many days, will A alone finish the work? 

(A) 30 days 

(B) 25 days 

(C) 26 days 

(D) 28 days 

Answer: A

 

Q. 55 If all the sides of a square are increased by 20%, then the area is increased by ………… 

(A) 46% 

(B) 44% 

(C) 43% 

(D) 45% 

Answer: B

 

Q. 56 If a shopkeeper gives two successive discounts of 20% and 15% on a book whose marked price is Rs. 850, then what is the selling price of the book? 

(A) ₹758 

(B) ₹587 

(C) ₹785 

(D) ₹578 

Answer: D

 

Q. 57 Amerchant claims that he sells his goods at CP. But uses a weight of 900g for the 1kg weight. find his gain % 

Answer: D

 

Q. 58 What is the least number which when increased by 8 is exactly divisible by 4, 5, 6 and 7? 

(A) 322 

(B) 312 

(C) 412 

(D) 422 

Answer: C

Explanation: 

LCM of 4, 5, 6 and 7 

= 420 

Therefore the required number is = 420 − 8 = 412 

Option Cis correct. 

 

Q. 59 Out of 12 terms, the average of first 6 terms is 36 and that of last 6 terms is 42. If the average of first 11 terms is 38, find the 12th term. 

(A) 44 

(B) 40 

(C) 50 

(D) 42 

Answer: C

Explanation: 

Total for 12 terms 

= (6 × 36) + (6 × 42) = 468 

Total for first 11 terms 

= 11 × 38 = 418 

= 468 − 418 = 50 

Therefore, 12th term 

Option Cis correct. 

 

Q. 60 Gold is 12 times as heavy as aluminum and copper is 5 times as heavy as aluminum.In what ratio gold and copper should be mixed to get an alloy 8 times of aluminum. 

(A) 2 : 1 

(B) 3 : 4 

(C) 1 : 2 

(D) 4 : 3 

Answer: B

 

Q. 61  Study the bar chart and answer the question based on it. 

In how many of the given years was the production of fertilizers less than the average production of the given years? 

(A)

(B)

(C)

(D)

Answer: B

 

Q. 62 Vishal covers a distance of 20 km in 30 min. If he covers half of the distance in 18 min, what should be his speed to cover the remaining distance and completes the whole journey in 30 min. 

(A) 50 km/hr 

(B) 55 km/hr 

(C) 60 km/hr 

(D) 65 km/hr 

Answer: A

 

Q. 63 Simplify: 

(A)

(B)

(C)

(D)

Answer: D

 

Q. 64 Om Prakash travels Bombay to Pune at a speed of 80 km/hr and returns back to Bombay by increasing his speed by 50%, then his average speed for the whole journey is …………. 

(A) 96 km/hr 

(B) 67 km/hr 

(C) 69 km/hr 

(D) 65 km/hr 

Answer: A

 

Q. 65 Manjeet singh sold an article for ₹9000 gaining 1/5 of its C.P. Find the gain %. 

(A) 10% 

(B) 22% 

(C) 20% 

(D) 25% 

Answer: C

 

Q. 66 Study the bar chart and answer the question based on it. 

The average production of 1999 and 2000 was less than the average production of which of the following pairs of years? 

(A) 1998 and 2000 

(B) 1996 and 1997 

(C) 2000 and 2001 

(D) 1995 and 2001 

Answer: C

 

Q. 67 If ₹3800 is distributed among A,B, C such that A: B= 1 : 2 and B: C= 3 : 5,the find the share of B. 

(A) ₹1200 

(B) ₹2000 

(C) ₹1400 

(D) ₹600 

Answer: A

Explanation: 

If ₹3800 is distributed among A,B, Csuch that A: B= 1 : 2 and B: C= 3 : 5 

Equating the two ratios we get A: B: C= 3 : 6 : 10 

Let x = 3800, A: B: C= 3x : 6x : 10x 

(A) + B+ C= 19x = 3800, x = 200 

(B) = 6 × 200 = 1200 

Option Ais correct. 

 

Q. 68 A cloth was 50 cm broad and 8 cm long. When washed, it was found to have lost 25% of its length and 14% of its breath. Then the percentage decreased in area is ………. 

(A) 34.5% 

(B) 35.5% 

(C) 36% 

(D) 35% 

Answer: B

 

Q. 69 In an election contest between A and B, A wins by the margin of 480 votes. If A gets 70% of the total votes. then total votes are 

(A) 1400 

(B) 1200 

(C) 4800 

(D) 1600 

Answer: B

 

Q. 70 The price of a scooter which was bought for ₹84,000 depreciates at the rate of 10% p.a. Find its price after 2 years? 

(A) ₹46080 

(B) ₹68040 

(C) ₹86040 

(D) ₹64800 

Answer: B

 

Q. 71 liters, milk and water are in the ratio 2:1. find the quantity of water to be added to make the ratio 4:3 

(A) 12 liters 

(B) 10 liters 

(C) 15 liters 

(D) 20 liters 

Answer: B

 

Q. 72 Acuboidal tank has 48000 liters of water. Find the depth of water in the tank if length of the tank is 6m and breadth is 4m. 

(A) 4 m 

(B) 6 m 

(C) 3 m 

(D) 2 m 

Answer: D

 

Q. 73 Find the number of years in which an amount invested at 8% p.a. simple interest doubles itself. 

(A) 12 years 

(B) 11 years 

(C) 12½ years 

(D) 10½ years 

Answer: C

 

Q. 74 The mean of a, b, c, d and e is 36. If the mean of b, d and e is 32, what is the mean of a and c? 

(A) 24 

(B) 44 

(C) 42 

(D) 46 

Answer: C

 

Q. 75 If the sum of five consecutive even numbers is 40 more than the average of those numbers, then find the middle number of the series? 

(A) 30 

(B) 10 

(C) 20 

(D) 40 

Answer: B

English 

Instructions 

For the following questions answer them individually 

Q. 76 Select the most appropriate option to substitute the underlined segment in the given sentence. If there is no need to substitute it, select No improvement. 

Your promotion will be depend the quality of your work. 

(A) depends upon 

(B) will be dependent 

(C) No improvement 

(D) depend on 

Answer: A

 

Q. 77 Select the word which means the same as the group of words given. 

A story, poem ora picture that can be interpreted to reveal a hidden meaning. 

(A) Allegory 

(B) Simile 

(C) Resemblance 

(D) Representation 

Answer: A

 

Q. 78 From the given options, identify the segment in the sentence which contains the grammatical error. An excellent response for his readers refreshes the author and stirs his creative sense and functional skills. 

(A) An excellent response 

(B) his creative sense 

(C) for his readers refreshes 

(D) functional skills 

Answer: C

 

Q. 79 Select the word which means the same as the group of words given. 

Concerning appreciation of art. 

(A) Particularistic 

(B) Idealistic 

(C) Poetic 

(D) Aesthetic 

Answer: D

 

Q. 80 Select the option that means the same as the given idiom. 

To bite off more than you can chew. 

(A) To eat very heavy and rich food 

(B) To challenge someone to do difficult work 

(C) To take on a task too big for oneself 

(D) To finish a work in bits and pieces 

Answer: C

 

Q. 81 Select the most appropriate option to fill in the blank. 

Online shopping platforms provide ………. information on books, groceries, medicines, etc.so that we can get the latest products. 

(A) timeworn 

(B) old 

(C) instant 

(D) previous 

Answer: C

 

Q. 82 Select the antonym of the given word. 

COMMENCE 

(A) Start 

(B) Pause 

(C) Close 

(D) Clamp 

Answer: C

 

Q. 83 Select the most appropriate word to fill in the blank. 

(A) software tool used to read electronic documents is known as …………….. 

(A) motherboard 

(B) browser 

(C) broadband 

(D) internet 

Answer: B

 

Q. 84 From the given options, identify the segment in the sentence which contains the grammatical error. In India, child labour is a socio – economic issue that need urgent attention by everyone. 

(A) child labour is a 

(B) socio – economic issue 

(C) by everyone. 

(D) that need urgent attention 

Answer: D

 

Q. 85 Select the most appropriate option to substitute the underlined segmenting the given sentence. If there is no need to substitute it, select No improvement. 

While Participating in the marathon an eighteen year old girl was knocked for by a passing truck. 

(A) No improvement 

(B) knocked up by 

(C) knocked in by 

(D) knocked down by 

Answer: D

Instructions 

In the following passage some words have been deleted.Fill in the blanks with the help of the alternatives given. 

Passage: 

Students should develop a quality of (1)…………. to elders and respecting their views. Their experience and knowledge can play a major role in shaping a student’s personality and career. Students should (2)…………… this much – needed knowledge and experience from parents and teachers. Being (3)………….. helps students to keep themselves morally high. One should not get carried away by success. Students should also learn to respect (4)…………. views and cope with diverse situations. These qualities will play a vital role in the (5)……………. of teamwork and leadership skills. 

 

Q. 86 Select the most appropriate option that will fill in the blank number 1. 

(A) listen 

(B) listened 

(C) listens 

(D) listening 

Answer: D

 

Q. 87 Select the most appropriate option that will fill in the blank number 2. 

(A) measure 

(B) acquire 

(C) access 

(D) assess 

Answer: B

 

Q. 88 Select the most appropriate option that will fill in the blank number 3. 

(A) smart 

(B) simple 

(C) intelligent 

(D) sharp 

Answer: B

 

Q. 89 Select the most appropriate option that will fill in the blank number 4. 

(A) general 

(B) expressive 

(C) typical 

(D) different 

Answer: D

 

Q. 90 Select the most appropriate option that will fill in the blank number 5. 

(A) structure 

(B) development 

(C) manufacture 

(D) arrangement 

Answer: B

Instructions 

For the following questions answer them individually 

 

Q. 91 Select the synonym of the given word. 

IMPLICIT 

(A) Tacit 

(B) Unclear 

(C) Honest 

(D) Unheard 

Answer: A

 

Q. 92 Select the most appropriate option to substitute the underlined segment in the given sentence.If there is no need to substitute it, select No improvement. 

The National Sample Survey is Directorate is first set up under the Ministry of Finance in 1950. 

(A) Directorate had first set up 

(B) Directorate were first set up 

(C) No improvement 

(D) Directorate was first set up 

Answer: D

 

Q. 93 Select the most appropriate word to fill in the blank. 

The news of the general elections was covered by ………… media. 

(A) sole 

(B) multiple 

(C) singular 

(D) single 

Answer: B

 

Q. 94 Select the antonym of the given word. 

EXPAND 

(A) Collect 

(B) Deposit 

(C) Express 

(D) Contract 

Answer: D

 

Q. 95 Select the correctly spelt word. 

(A) Audibal 

(B) Audeuble 

(C) Audible 

(D) Audiable 

Answer: C

 

Q. 96 Select the most appropriate option to fill in the blank. 

People high in humility avoid expecting ………… special treatment. 

(A) none 

(B) any 

(C) one 

(D) only 

Answer: B

 

Q. 97 Select the option that means the same as the given idiom. 

Bark up the wrong tree. 

(A) To make no mistakes about doing something 

(B) To make dogs bark at cats hiding in trees 

(C) To send dogs to track someone 

(D) To be wrong about the reason for or way of doing something 

Answer: D

 

Q. 98 From the given options, identify the segment in the sentence which contains the grammatical error. Amere 43% of Indian woman own cellular phones, as opposed to almost 80% of men. 

(A) as opposed to 

(B) own cellular phones 

(C) A mere 43% of Indian woman 

(D) almost 80% of men 

Answer: C

 

Q. 99 Select the synonym of the given word. 

RIDDLE 

(A) Puzzle 

(B) Response 

(C) Game 

(D) Reaction 

Answer: A

 

Q. 100 Select the correctly spelt word. 

(A) Vegetaranian 

(B) Vegaitarian 

(C) Vegetarian 

(D) Vezetarian 

Answer: C

SSC GD 3 March 2019 Shift-I Previous Year Paper

SSC GD 3rd March 2019 Shift-1 

Reasoning 

Instructions 

For the following questions answer them individually 

Q. 1 Select the Venn diagram that best illustrates the relationship between the three given classes. Keyboard, Monitor, Mouse 

Answer: (B) 

 

Q. 2 Find the missing number from the below options. 

(A) 12 

(B) 48 

(C) 24 

(D) 56 

Answer: (C) 

Explanation: 

Answer is 24. 

 

Q. 3 Choose the option that most closely resembles the mirror image of the given figure when mirror is placed at right side. 

Answer: (B) 

Explanation: 

Option (B) is correct Option. 

 

Q. 4 Select the option that is related to the fourth term in the same way as the first term is related to the second term. 

MNQR : LMPQ :: ? : RSVW 

(A) STWX 

(B) WYTS 

(C) SVWY 

(D) VIWY 

Answer: (A) 

Explanation: 

MNQR : LMPQ :: ? : RSVW 

M-1=L 

N-1=M 

Q-1=P 

R-1=Q 

Similarly, 

S-1=R 

T-1=S 

W-1=V 

X-1=W 

STWX : RSVW 

 

Q. 5 If RED is coded as 19, then how WED would be coded? 

(A) 21 

(B) 28 

(C) 22 

(D) 24 

Answer: (D) 

 

Q. 6 A, B, C, D, E, F, G and H are sitting around a round table facing the centre. D is fourth to the right of H and second to the left of B. F is fourth to the right of B. C is fourth to the right of E, whois not an immediate right of B or D. A is not an immediate neighbour of D. In which of the following combinations is the third person sitting in between the first and the second persons? 

(A) CBA 

(B) ABC 

(C) GCD 

(D) AHE 

Answer: (C) 

 

Q. 7 Select the option that is related to the third term in the same wayas the second term is related to the first term. 

Television : Telecast :: Radio : ? 

(A) Broadcast 

(B) Friendship 

(C) Compose 

(D) Talking 

Answer: (A) 

Explanation: 

Television is medium to Telecast 

Similarly, Radio is used for Broadcast. 

 

Q. 8 In a certain code “SMART” is written as “RKXNO”, How is “GREAT” written in that code? 

(A) BGFWO 

(B) FPBWO 

(C) RPFGW 

(D) FPBWR 

Answer: (B) 

Explanation: 

We go in reverse alphabet series to get logic of this word “SMART” = “RKXNO”, 

S-1=R 

M-2=K 

A-3=X 

R-4=N 

T-5=O 

Similary, “GREAT” = “FPBWO” 

G-1=F 

R-2=P 

E-3=(B) 

A-4=W 

T-5=O 

Option (B) is correct. 

 

Q. 9 Choose the option which completes the given figure series 

Answer: (D) 

 

Q. 10 Select the option that is related to the Third term in the same way as the first term is related to the second term. 

42 : 72 :: 12 : ? 

(A) 30 

(B) 43 

(C) 35 

(D) 48 

Answer: (A) 

 

Q. 11 Select the option that is related to the third term in the same way as the first term is related to the second term. 

Mango : Fruit :: Potato : ? 

(A) Flower 

(B) Fruit 

(C) Vegetable 

(D) Stem 

Answer: (C) 

Explanation: 

Mango is a Fruit. Similarly, Potato is an vegetable. 

 

Q. 12 Four words have been given out of which three are alike in some manner, while one is different. Choose the odd one. 

(A) Dumb 

(B) Sinus 

(C) Deaf 

(D) Blind 

Answer: (B) 

Explanation: 

Dumb,Deaf,Blind are Deficiency in human body hence they are similar and form a one group. But Sinus is a disease related to breathing problem. 

Hence Sinus is odd one. 

 

Q. 13 Sonam is older than Renu. komal is younger than Renu. Priya is older than Sonam. Who is the eldest of them? 

(A) Sonam 

(B) Renu 

(C) Priya 

(D) Komal 

Answer: (C) 

Explanation: 

1)Sonam is older than Renu. 

=Sonam >Renu 

2) Komal is younger than Renu. 

=Renu > Komal 

3)Priya is older than Sonam. 

=Sonam >Priya 

If we consolidate all three sentences 1,2,3 the Correct Sequence of age from oldest to youngest is Priya > Sonam > Renu > Komal . Hence Priya is oldest in age. 

 

Q. 14 Four options have been given out of which three are alike in some manner, while one is different. Choose the odd one. 

(A) 2248 

(B) 3036 

(C) 1258 

(D) 3196 

Answer: (D) 

Explanation: 

Add first three digit to get fourth digit of number. 

2248(2 + 2 + 4 = 8) 

3036(3 + 0 + 3 = 6) 

1258(1 + 2 + 5 = 8) 

3196(3 + 1 + 9 = 13 13 6 

Odd one is Option (D) as it does not follow the pattern, last digit should be instead of ) 

 

Q. 15 The statements below are followed by two conclusions labeled I and II. Assuming that the information in the statements is true, even if it appears to be at variance with generally established facts, decide which conclusion(s) logically and definitely follow(s) from the information given in the statements. 

Statements: 

All roses are fruits. 

All fruits are trees. 

Conclusions: 

I. All trees are roses. 

II. Some fruits are roses. 

(A) Both Conclusions I and II follow 

(B) Either Conclusion I or II follows 

(C) Only Conclusion I follows 

(D) Only Conclusion II follows 

Answer: (D) 

 

Q. 16 Select the correct option that will fill in the blank and complete the series. 

14, 18, 24, 32, 42, ? 

(A) 54 

(B) 50 

(C) 52 

(D) 48 

Answer: (A) 

Explanation: 

18-14 = +4 

24-18 = +6 

32-24 = +8 

42-32 = +10 

Hence last number 42 +12 = 54 

14, 18, 24, 32, 42, 54 

 

Q. 17 Four letter clusters have been given out of which three are alike in some manner, while one is different. Choose the odd one. 

(A) KMP 

(B) ACF 

(C) TVY 

(D) BCG 

Answer: (D) 

Explanation: 

K+2=M , M+3=P (KMP) 

A+2=(C) , C+3=F (ACF) 

T+2=V , V+3=F (TVY) 

but, 

B+1=D, C+4=G (BCG) is not the correct pattern. 

Hence Option (D) is odd 

 

Q. 18 The statements below are followed by two conclusions labeled I and II. Assuming that the information in the statements is true, even if it appears to be at variance with generally established facts, decide which conclusion(s) logically and definitely follow(s) from the information given in the statements. 

Statements: 

Some grapes are mangoes. 

All mangoes are apples. 

Conclusions: 

I. Some mangoes are grapes. 

II. All apples are grapes. 

(A) Only Conclusion I follows 

(B) Either Conclusion I or II follows 

(C) Both Conclusions I and II follow 

(D) Only Conclusion II follows 

Answer: (A) 

 

Q. 19 Select the correct option that will fill in the blank and complete the series. 

117, 107, 92, 72, ……… 

(A) 42 

(B) 47 

(C) 37 

(D) 52 

Answer: (B) 

Explanation: 

117-10 = 107 

107- 15 = 92 

92- 20 = 72 

72 – 25= 47 

Hence, Correct Sequence will be 117, 107, 92, 72, 47 

 

Q. 20 Choose the option figure in which the problem figure is hidden/embedded. 

Answer: (D) 

 

Q. 21 Select the correct option that will fill in the blank and complete the series. 

A, G, L, P, S, … 

(A)

(B)

(C)

(D)

Answer: (B) 

Explanation: 

Logic of Q. is A+6 =G, G+5=L , L+4=P, P+3=S, S+2=U, Hence U is correct option. 

 

Q. 22 P, M, D, A, F, H, R and B are sitting around a circle facing at the center. R is fourth to the right of A, who is third to the right of P. M is second to the left of H, who is second to the left of P. D is third to the right of B. 

Which of the following pairs are immediate neighbors of B? 

(A) AP 

(B) MP 

(C) AF 

(D) FM 

Answer: (C) 

 

Q. 23 A piece of paper is folded and cut as shown below in the question figures. From the given answer figures, how will it appear when opened? 

Answer: (A) 

 

Q. 24 If ‘ + ’ means ‘ × ’, ‘ − ’ means ‘ + ’, ‘ × ’ means ‘ ÷ ’ and ‘ ÷ ’means ‘ − ’ , then find the value of the following equation:- 

21 ÷ 8 + 2 − 12 × 3 =? 

(A) 13.5 

(B) 14 

(C) 10 

(D)

Answer: (D) 

 

Q. 25 Which of the following answer figure will complete the given figure? 

Answer: (C) 

General knowledge

Instructions 

For the following questions answer them individually 

Q. 26 The Supreme Court was established at Fort William in …….. as the Apex Court in 1774. 

(A) Delhi 

(B) Shimla 

(C) Mumbai 

(D) Kolkata 

Answer: (D) 

 

Q. 27 Who won the 2018 US Open Men’s Tennis? 

(A) Andy Murray 

(B) Rafael Nadal 

(C) Novak Djokovic 

(D) Roger Federer 

Answer: (C) 

 

Q. 28 The Union Budget 2018 allocated ……… for food processing, which was almost double of the allocation in the previous year’s budget. 

(A) ₹2,900 crore 

(B) ₹1,400 crore 

(C) ₹1,900 crore 

(D) ₹2,400 crore 

Answer: (B) 

 

Q. 29 In the 2018 Sultan Azlan Shah Cup, India beat which country 4-1 to comeat the fifth place? 

(A) Australia 

(B) Malaysia 

(C) Ireland 

(D) Pakistan 

Answer: (C) 

 

Q. 30 Who did Mughal Emperor Babur defeat in the Battle of Ghagra in 1529? 

(A) Yusuf Adil Shah 

(B) Mahmud Lodi 

(C) Dilawar Khan Husain 

(D) Qasim Barid I 

Answer: (B) 

 

Q. 31 The mandible is part of the ………… 

(A) hand 

(B) arm 

(C) skull 

(D) pelvic girdle 

Answer: (C) 

 

Q. 32 ………. is an important copper ore mineral 

(A) Galena 

(B) Cassiterite 

(C) Cinnabar 

(D) Chacocite 

Answer: (D) 

 

Q. 33 The GDP growth rate for the fiscal year 2017-2018 was pegged at ……… by the Economic Survey report 2017-18. 

(A) 8.25% 

(B) 7.25% 

(C) 5.75% 

(D) 6.75% 

Answer: (D) 

 

Q. 34 Which of the following is the southernmost of the Lakshadweep islands? 

(A) Bitra 

(B) Kavaratti 

(C) Amini 

(D) Minicoy 

Answer: (D) 

 

Q. 35 Name the type of folk songs of Sikkim which describe the natural beauty of the state. 

(A) Gha To Kito 

(B) Mando 

(C) Khubakeshei 

(D) Deknni 

Answer: (A) 

 

Q. 36 Studies on political and social inequalities in democracy show that: 

(A) Dictatorship is better than democracy 

(B) Inequalities do not exist in dictatorships 

(C) Inequalities exist in democracies 

(D) Democracy deters development 

Answer: (C) 

 

Q. 37 Name the first Indian to become a member of the British Parliament. 

(A) Piloo Mody 

(B) Dadabhai Naoroji 

(C) Bhikaiji Cama 

(D) Jamshetji Tata 

Answer: (B) 

 

Q. 38 Which of the following dances is a dance form from Madhya Pradesh? 

(A) Lezim 

(B) Chiraw 

(C) Ottam Thullal 

(D) Macha 

Answer: (D) 

 

Q. 39 Who founded the Maurya dynasty after defeating Dhana Nanda? 

(A) Bindusara 

(B) Ashoka 

(C) Kunala 

(D) Chandragupta 

Answer: (D) 

 

Q. 40 Weber is the unit of ………… 

(A) electric conductance 

(B) magnetic flux 

(C) magnetic flux density 

(D) capacitance 

Answer: (B) 

 

Q. 41 The formula of Acetonitrile is ………… 

(A) CH3OCN 

(B) C2N2 

(C) CH3CN 

(D) CH2CHCN 

Answer: (C) 

 

Q. 42 Which of the following is a correct equation of motion? 

(A) u = v + at2 

(B) 2s = (v − u)t 

(C) 2s = 2ut + at2 

(D) v2 + u2 = 2as 

Answer: (C) 

 

Q. 43 Which of the following Indian National political parties is reputed to be one of the oldest, founded in 1885 and has experienced many splits? 

(A) Communist Party of India — Marxist (CPI-M) 

(B) Nationalist Congress Party (NCP) 

(C) Indian National Congress (INC) 

(D) Bahujan Samaj Party (BSP) 

Answer: (C) 

 

Q. 44 Losar Festivalis celebrated by the people of the ……….. tribe. 

(A) Khasi 

(B) Kodava 

(C) Munda 

(D) Monpa 

Answer: (D) 

 

Q. 45 Ganymede is by far the largest moon in our solar system orbits around the planet …….. 

(A) Uranus 

(B) Neptune 

(C) Jupiter 

(D) Saturn 

Answer: (C) 

 

Q. 46 Whose birthday is celebrated as Engineer’s Day? 

(A) Abdul Kalam Azad 

(B) J R D Tata 

(C) M Visvesvaraya 

(D) C V Raman 

Answer: (C) 

 

Q. 47 Which of the following rivers flows into the Arabian Sea? 

(A) Mahi 

(B) Krishna 

(C) Penna 

(D) Rushikulya 

Answer: (A) 

 

Q. 48 Which book by Kiran Desai is a tale of a young man named Sampath Chawla? 

(A) Hullabaloo in the Guava Orchard 

(B) Selection Day 

(C) An Equal Music 

(D) It Happens for a Reason 

Answer: (A) 

 

Q. 49 Minimum Support Price for 2018-19 for Paddy of the Common variety is set at ………. 

(A) ₹2,430 per quintal 

(B) ₹1,750 per quintal 

(C) ₹5,150 per quintal 

(D) ₹1,950 per quintal 

Answer: (B) 

 

Q. 50 Which is the first ever known autobiography of an Indian woman? 

(A) My reminences 

(B) Beyond the jungle 

(C) Amar Jiban 

(D) Cityty of 2 gateways 

Answer: (C) 

Quant 

Instructions 

For the following questions answer them individually 

Q. 51 Simplify : 

5 − 6.5 ÷ 13 + 2.3 × 0.8 + 0.4 

(A) 6.83 

(B) 6.74 

(C) 7.38 

(D) 5.38 

Answer: (B) 

 

Q. 52 A car travels 60km in the first hour , 65km in the second hour , 70km in the third hour, then the average speed of the car is 

(A) 66 km/hr 

(B) 62 km/hr 

(C) 56 km/hr 

(D) 65 km/hr 

Answer: (D) 

Explanation: 

Average speed=Total distance/Total time 

Distance 1=60*1=60 km 

Distance 2=65*1=65 km 

Distance 3=70*1=70 km 

Total distance=195 km 

Total time=3 hours 

Average speed=195/3 

=65 km/hr 

 

Q. 53 Amit borrowed ₹8000 from a moneylender with Simple interest for as many years as the rate of interest. If he paid ₹2000 as interest at the end of loan period, what was the rate of interest? 

(A) 5% 

(B) 7% 

(C) 6% 

(D) 8% 

Answer: (A) 

Explanation: 

I=PTR/100 

Given I=R 2000 

2000=8000*R*R/100 

R*R=2000/80 

R*R=5*5 

R=5% 

 

Q. 54 Average of ‘n’ observations is 38, average of ‘n’ other observations is 42 and average of remaining ‘n’ observations is 55. Average of all the observations is: 

(A) 45 

(B) 35 

(C) 55 

(D) 40 

Answer: (A) 

Explanation: 

Sum of first n observations =38n 

Sum of next n observations=42n 

Sum of last n observations=55n 

Total sum=135n 

Average=135n/3n =45 

 

Q. 55 I sold an article for ₹8800 thus gained 10%. For what amount should sell it to get 15% profit? 

(A) ₹9,400 

(B) ₹9,300 

(C) ₹9,200 

(D) ₹9,500 

Answer: (C) 

Explanation: 

SP=8800 

Profit percent=10% 

Therefore 1.1*CP=8800 

CP=8800*100/11 

CP=8000 

We have profit percent=15 

SP=1.15CP 

SP=1.15*8000 

SP=Rs 9200 

 

Q. 56 Simplify: 8 + 3 − ( 5/2 × ⅓) of 12/5 + 4/3 × 3/8

(A) 9.5 

(B) 10 

(C)

(D) 19 

Answer: (A) 

 

Q. 57 Study the following line graph and answer the question based on it. 

What is the difference between the total productions of the two companies from 1997 to 2000? 

(A) 40000 

(B) 50000 

(C) 30000 

(D) 60000 

Answer: (B) 

Explanation: 

Sum of vehicles manufactured in the give years by X =78+119+99+141=437 

Sum of vehicles manufactured in the given years by Y =139+120+100+128=487 

Difference =487k-437k 

=50k 

=50000 

 

Q. 58 A, B and C can do a piece of work in 12 , 15 and 30 days respectively. If they do a work together, find the ratio in which the amount is to be distributed 

(A) 4 : 5 : 2 

(B) 2 : 5 : 4 

(C) 2 : 4 : 5 

(D) 5 : 4 : 2 

Answer: (D) 

Explanation: 

LCM of 12,15 and 30 is 60 units 

Each day (A) can do 60/12 =5 units of work 

Each day (B) can do 60/15 =4 units of work 

Each day (C) can do 60/30 =2 units of work 

So ratio of money is in the ratio of units of work done each day 

i.e 5:4:2 

 

Q. 59 Study the following line graph and answer the question based on it. 

In which of the following years, the difference between the productions of Companies X and Y was the minimum among the given years? 

(A) 1997 

(B) 2000 

(C) 2002 

(D) 1999 

Answer: (C) 

Explanation: 

Difference between the sales in 1997=139-119=20 

Difference between the sales in 1998=120-99=21 

Difference between the sales in 1999=141-100=41 

Difference between the sales in 2000=128-78=50 

Difference between the sales in 2001=120-107=13 

Difference between the sales in 2002=159-148=11 

So in the year 2002 it is least 

 

Q. 60 Ramu got a new table for 25% discount. Had he got no discount, Ramu would have had to pay ₹185 more. How much did Ramu pay for the table? 

(A) 535 

(B) 545 

(C) 555 

(D) 565 

Answer: (C) 

 

Q. 61 In an examination, a student has to secure 35% of the maximum marks to pass. One student got 70 marks and failed by 70 marks. The maximum marks in the examination are 

(A) 400 

(B) 300 

(C) 500 

(D) 200 

Answer: (A) 

 

Q. 62 The length and breadth of a rectangle are in the ratio 8:3. Find Its area if perimeter is 132 cm. 

(A) 864 cm2 

(B) 684 cm2 

(C) 648 cm2 

(D) 846 cm2 

Answer: (A) 

 

Q. 63 Two cylindrical cans has the base of the same size. The diameter of each is 14cm. The height of one can is 10cm and another 20cm. Find the ratio of their volumes. 

(A) 3:2 

(B) 2:3 

(C) 1:2 

(D) 2:1 

Answer: (C) 

 

Q. 64 The ratio between two natural number is 3:4 , find the smallest number which must be added to both the numbers so that the ratio becomes 4:5 and the sum of the numbers after adding the smallest number will be 27. 

(A)

(B)

(C)

(D)

Answer: (D) 

 

Q. 65 Study the following line graph and answer the question based on it. 

The production of Company Y in 1998 was approximately what percent of the production of Company X in the same year 

(A) 212.1% 

(B) 211.2% 

(C) 112.2% 

(D) 122.2% 

Answer: (D) 

Explanation: 

The production of Company Y in 1998 is 99k 

The production of Company X in 1998 is 121k 

99k=100% 

121k=(121000/99000)*100 

=122.22% 

 

Q. 66 A alone can do a piece of work in 12 days and B alone in 16 days. They under took to do the work for ₹350. How much will B get ? 

(A) ₹150 

(B) ₹200 

(C) ₹250 

(D) ₹180 

Answer: (A) 

Explanation: 

A can do the work in 12 days and (B) can do in 16 days 

LCM of 12 and 16 is 48 units 

‘A’ can do 48/12 =4 units each day 

‘B’ can do 48/16 =3 units each day 

Each day they can do 7 units of work. 

So total days=48/7 days 

(B) does 3*(48/7) units of the total units 

So (3*(48/7)/48)*350=Rs 150 

 

Q. 67 The mean of a set of eight numbers is 63. If one of the numbers is discarded, the mean of the remaining numbers is 58. The value of discarded number is 

(A) 88 

(B) 89 

(C) 78 

(D) 98 

Answer: (D) 

Explanation: 

Sum of the eight numbers=63*8 

=504 

Sum of the 7 numbers =58*7 

=406 

The discarded number=504-406 

=98 

 

Q. 68 Average of first eight multiples of 8 is 

(A) 36 

(B) 46 

(C) 32 

(D) 72 

Answer: (A) 

Explanation: 

First 8 multiples of 8 are 8,16….64 

Sum=8+16….64 

Sum=8(1+2…8) 

=8(8)(9)/2 

=288 

Average=288/8 

=46 

 

Q. 69 A shop keeper reduces the selling price of an article by 39, a gain of 10% turns in to a loss of 3%, then the original price of the article is? 

(A) ₹350 

(B) ₹300 

(C) ₹250 

(D) ₹200 

Answer: (B) 

Explanation: 

Here the net change is 10+3 =13% 

Given that this 13% change has occurred due to the change in SP i.e SP has decreased by 39 

So we can say that 13% of CP=39 

13*CP/100 =39 

CP=3900/13 

CP=Rs 300 

 

Q. 70 The differences between the circumference and diameter of a circle is 60 cm, then radius is 

(A) 12 cm 

(B) 13 cm 

(C) 14 cm 

(D) 15 cm 

Answer: (C) 

Explanation: 

2πr − 2r 

Given =60 

2r(π − 1) 

=60 

2r*15=60*7 

r=14 cm 

 

Q. 71 The population of Varanasi is 2,25,000. If the annual birth rate and death rate are 9 % and 7 %, calculate the population of city after 2 years. 

(A) 432090 

(B) 423900 

(C) 243090 

(D) 234090 

Answer: (D) 

Explanation: 

In this case the net increase per year is (9-7)%=2%. 

so after 1 year the increase will be 225000*1.02=229500 

After 1 more year the increase will be 229500*1.02=234090 

 

Q. 72 The ratio between 2 numbers is 4:5 and the sum of their squares is 1025 , then the numbers are 

(A) 20 and 25 

(B) 24 and 30 

(C) 8 and 10 

(D) 40 and 50 

Answer: (A) 

Explanation: 

Let the numbers be 4x and 5x 

Sum of the squares is 1025 

(4x)2 + (5x)2 

=1025 

41x2 

=1025 

x2 

=25 

x=5 

The numbers are 20 and 25 

 

Q. 73 A car travels a certain distance at a speed of 120km/hr in 4 hours. What should be the speed of the car if the car covers the same distance in 3 hours 20 minutes. 

(A) 142 km/hr 

(B) 140 km/hr 

(C) 124 km/hr 

(D) 144 km/hr 

Answer: (D) 

Explanation: 

Distance=Speed*time 

=120*4 

=480 km 

Speed=distance/time 

=480/(10/3) 

=144 km/hr 

 

Q. 74 Three persons A, B and C can complete one round of circular track in 8, 15 and 20 minutes respectively. If they start together at 8 A.M from a certain point, at which time will they be together again at the starting point? 

(A) 10 AM 

(B) 12 Noon 

(C) 11 AM 

(D) 9 AM 

Answer: (A) 

 

Q. 75 The ratio of the ages of Arun and Anand is 3:1. After 5 years Arun age will be 5 more than two times of Anand’s age. Then the present age of Arun is 

(A) 40 

(B) 30 

(C) 35 

(D) 25 

Answer: (B) 

Explanation: 

Let the ages of Arun and Anand be 3x and x 

After 5 years Arun’s age=3x+5 and Anand’s age=x+5 

Given 3x+5=2(x+5)+5 

3x+5=2x+10+5 

x=10 

Present age of Arun=30 years 

English 

Instructions 

For the following questions answer them individually 

Q. 76 Select the most appropriate option to substitute the underlined segment in the given sentence. If there is no need to substitute it, select No improvement. 

He welcomed the audience with a heart felt gratitude for their kind presence on the day of the social gathering in the college. 

(A) No Improvement 

(B) for their kind presences 

(C) of their kind presence 

(D) for their kind present 

Answer: (A) 

 

Q. 77 Select the most appropriate option to substitute the underlined segment in the given sentence.If there is no need to substitute it, select No improvement. 

My cousin won’t be able to leave for Mumbai until his parents will have arrived. 

(A) until his parents will arrive 

(B) until his parents arrive 

(C) until his parents won’t arrive 

(D) No Improvement 

Answer: (B) 

 

Q. 78 Select the most appropriate option to fill in the blank. 

There is no prediction that ………… how fast dehydration can affect your general health. 

(A) have been showing 

(B) have show 

(C) shows 

(D) showed 

Answer: (C) 

 

Q. 79 Select the synonym of the given word. 

FLIMSY 

(A) general 

(B) coarse 

(C) sturdy 

(D) Feeble 

Answer: (D) 

 

Q. 80 From the given options, identify the segment in the sentence which contains the grammatical error. The researchers have discovered numerous sample of decorated sandstone, including chapel art depicting the Egyptian goddess. 

(A) researchers have discovered 

(B) including chapel art 

(C) depicting the Egyptian goddess 

(D) numerous sample of decorated sandstone 

Answer: (D) 

 

Q. 81 Select the most appropriate option to fill in the blank. 

Leadership …………. a rational approach to achieve success in any task. 

(A) requiring 

(B) require 

(C) required 

(D) requires 

Answer: (D) 

 

Q. 82 Select the word which means the same as the group of words given. 

Incapable of failure or error. 

(A) Incorrigible 

(B) Incurable 

(C) Infallible 

(D) Irredeemable 

Answer: (C) 

 

Q. 83 Select the correctly spelt word. 

(A) Miscelleneous 

(B) Miscellaneous 

(C) Miscellaneius 

(D) Miscelaneous 

Answer: (B) 

 

Q. 84 From the given options, identify the segment in the sentence which contains the grammatical error. In the modern world, the computer is an amazing gift to mankind for doing two thing effortlessly. 

(A) the computer is 

(B) an amazing gift 

(C) In the modern world 

(D) for doing two thing effortlessly. 

Answer: (D) 

 

Q. 85 Select the most appropriate word to fill in the blank. 

Humans have a natural …………. to coordinate their movements with music. 

(A) disability 

(B) direction 

(C) order 

(D) tendency 

Answer: (C) 

Instructions 

In the following passage some words have been deleted. Fill in the blanks with the help of the alternatives given. Passage: 

The concept of sustainable development provides a (1)……….. for the integration of environmental policies and development strategies. Sustainable development (2)……….. to meet the needs and aspirations of the present without compromising the (3)………… to meet those in the future. In other words, sustainable development is a (4)………… of change in which the exploitation of resources, the direction of investments, technological development and institutional changes are all in harmony and (5)…………… both current and future potential to meet 

human needs and objectives. 

 

Q. 86 Select the most appropriate option that will fill in the blank number 1. 

(A) comparison 

(B) framework 

(C) figure 

(D) provision 

Answer: (B) 

 

Q. 87 Select the most appropriate option that will fill in the blank number 2. 

(A) diverts 

(B) seeks 

(C) rejects 

(D) distracts 

Answer: (B) 

 

Q. 88 Select the most appropriate option that will fill in the blank number 3. 

(A) invention 

(B) operation 

(C) production 

(D) orientation 

Answer: (D) 

 

Q. 89 Select the most appropriate option that will fill in the blank number 4. 

(A) progress 

(B) permission 

(C) process 

(D) sample 

Answer: (C) 

 

Q. 90 Select the most appropriate option that will fill in the blank number 5. 

(A) dismiss 

(B) enhance 

(C) resist 

(D) relieve 

Answer: (B) 

Instructions 

For the following questions answer them individually 

 

Q. 91 Select the synonym of the given word. 

BREACH 

(A) Restriction 

(B) Violation 

(C) Settlement 

(D) Restraint 

Answer: (B) 

 

Q. 92 Select the most appropriate option to substitute the underlined segment in the given sentence. If there is no need to substitute it, select No improvement. 

Some people are so good that a mere looks at them pleases you lot. 

(A) No Improvement 

(B) a mere look at their pleases you 

(C) a mere looks at them pleases you 

(D) a mere look at them pleases you 

Answer: (D) 

 

Q. 93 Select the most appropriate word to fill in the blank. 

There is a high degree of ………… from e-mail based to social-media based messaging globally. 

(A) transition 

(B) process 

(C) transaction 

(D) method 

Answer: (A) 

 

Q. 94 Select the word which means the same as the group of words given. 

A cold – blooded vertebrate typically living on land but breeding in water. 

(A) Fish 

(B) Amphibians 

(C) Mammals 

(D) Reptiles 

Answer: (B) 

 

Q. 95 Select the antonym of the given word. 

LOGICAL 

(A) Practical 

(B) Contradictory 

(C) Sensible 

(D) Useful 

Answer: (B) 

 

Q. 96 Select the option that means the same as the given idiom. 

Apples and Oranges. 

(A) Two unlike things or people 

(B) People working together on a project 

(C) People with similar approaches 

(D) Two similar situations with likely results 

Answer: (A) 

 

Q. 97 From the given options, identify the segment in the sentence which contains the grammatical error. A clear and defined perception enabled her to dismissed the suggestion given by some people around her. 

(A) suggestion given by 

(B) clear and defined 

(C) some people around her. 

(D) enabled her to dismissed 

Answer: (D) 

 

Q. 98 Select the antonym of the given word. 

PAUCITY 

(A) Scarcity 

(B) Surplus 

(C) Availability 

(D) Shortage 

Answer: (B) 

 

Q. 99 Select the correctly spelt word. 

(A) Refrigirator 

(B) Refreegerator 

(C) Refrigerator 

(D) Refridgerator 

Answer: (C) 

 

Q. 100 Select the option that means the same as the given idiom. 

To burn the midnight oil. 

(A) To eat something at midnight 

(B) To use oil for burning a lamp 

(C) To work till very late in the night 

(D) To fight with someone at midnight 

Answer: (C) 

SSC GD 1 March 2019 Shift-II Previous Year Paper

SSC GD 1st March 2019 Shift-II 

Reasoning 

Instructions 

For the following questions answer them individually 

Q. 1 Select the option that is related to the third term in the same way as the second term is related to the first term.

Man : Mammal :: Butterfly : ……. 

(A) Caterpillar 

(B) Insect 

(C) Bird 

(D) Reptile 

Answer: (B) 

Explanation: 

Man previous history form was known as Mammal. 

Similarly, before becoming a Butterfly it is known as Insect. 

 

Q. 2 Select the option that is related to the third term in the same way as the second term is related to the first term. 

KN : 121/196 : : RU:______ 

(A) 256/400 

(B) 256/196 

(C) 324/144 

(D) 324/441 

Answer: (D) 

 

Q. 3 If SCIENCE is coded as 14 and MATHS is coded as 10, then what will be the code for EXAMINATION? 

(A) 21 

(B) 22 

(C) 11 

(D) 23 

Answer: (B) 

 

Q. 4 A square transparent sheet with a pattern is given. Select from among the four alternatives how the pattern would appear when the transparent sheet is folded at the dotted line. 

Answer: (B) 

Explanation: 

Option (B) is correct Option when folded perfectly. 

 

Q. 5 Select the number-pair that is different from the rest. 

(A) 7-49 

(B) 16-256 

(C) 12-144 

(D) 13-196 

Answer: (D) 

Explanation: 

72 = 49 

162 = 256 

122 = 144 

132 = 169 

Hence Odd one is . But in question 196 is given. 

Hence Option (D) is odd one. 

 

Q. 6 Six friends, Anitha, Punitha, Chithra, Deepthi, Harini and Vani, are sitting in a field. Anitha and Punitha are from the Green house, where as the rest belong to the Red house. Deepthi and Vani are tall. where as the others are short. Anitha, Chithra and Deepthi are wearing saris. where as the others are wearing frocks. 

Who among them is tall and from the Red house but not wearing a sari? 

(A) Deepthi 

(B) Anitha 

(C) Vani 

(D) Chithra 

Answer: (C) 

Explanation: 

1) Anitha and Punitha are from the Green house. 

2) where as the rest belong to the Red house.( i.e. Chithra, Deepthi, Harini and Vani) 

3) Person Wearing sari are i.e. Anitha, Chithra and Deepthi 

4) Whereas other are wearing frocks. (Other than sari person.. place in this frock box) 

5) Deepthi and Vani are tall. Other are short. (We will denote with Tall in capital words and short in small alphabets.) 

6) Now we need to find Who among them is tall and from the Red house but not wearing a sari. (i.e. we will cancel the person who is short and person wearing frock.) 

Hence, Vani is from Red house but not wearing a sari. 

 

Q. 7 Read the given statements and conclusion carefully and decide which of the following options is true with respect to the given conclusion. 

Statements: 

Men who are handsome are generally of the intellectual type. 

Vikram is handsome. 

Conclusion: 

Vikram is intellectual. 

(A) Probably false 

(B) Probably true 

(C) True 

(D) False 

Answer: (B) 

Explanation: 

Conclusion is True – Vikram is intellectual. Hence Option (B) is correct. 

 

Q. 8 Which two signs/numbers need to be interchanged to make the given equation correct? 

8 ÷ 2 + 4 = 10 

(A) ÷ and +, 8 and 4 

(B) ÷ and + 

(C) + and ÷, 8 and 2 

(D) + and ÷, 2 and 4 

Answer: (D) 

 

Q. 9 Select the option in which the given figure is embedded. 

Answer: (B) 

 

Q. 10 Choose the option that most closely resembles the mirror image of the given word when mirror is placed at right side. 

Answer: (A) 

 

Q. 11 The statements below are followed by two conclusions labeled I and II. Assuming that the information in the statements is true, even if it appears to be at variance with generally established facts, decide which conclusion(s) logically and definitely follow(s) from the information given in the statements. 

Statements: 

Some girls are women. 

No woman is brown. 

Conclusions: 

I. Some girls are not brown. 

II. Some women are girls. 

(A) Only conclusion II follows. 

(B) Only conclusion I follows. 

(C) Both conclusions I and II follow. 

(D) Neither conclusion I nor II follows. 

Answer: (C) 

 

Q. 12 Seven friends — Ajith, Bala, Hari, David, Raja, Sekar and Guna — are sitting around a circular table. David is not the neighbour of Hari and Raja. Ajith is the neighbour of Bala and Hari. Guna, who is second to the left of David, is the neighbour of Sekar and Raja. What is the position of Hari? 

(A) To the immediate left of Ajith 

(B) To the immediate right of Raja 

(C) Third to the left of Sekar 

(D) Second to the left of Bala 

Answer: (C) 

 

Q. 13 Select the number that will come next in the following series. 

1, 5, 9, 17, 25, 37, 49, 65, 81, ……… 

(A) 99 

(B) 110 

(C) 100 

(D) 101 

Answer: (D) 

Explanation: 

1-5=4 

5-9=4 

9-17=8 

17-25=8 

25-37=12 

37-49=12 

49-65=16 

65-81=16 

Hence,81+20= 101. 

1, 5, 9, 17, 25, 37, 49, 65, 81, 101… 

 

Q. 14 Four words have been given out of which three are alike in some manner, while one is different. Choose the odd one. 

(A) Cub 

(B) Sty 

(C) Hive 

(D) Kennel 

Answer: (A) 

Explanation: 

Sty is house of a Pig 

Hive is house of HoneyBee 

Kennel is house of Dog. 

But, Cub is a small child fox,bear,lion. Hence Option (A) is odd one. 

 

Q. 15 Select the word-pair that is analogous to the given word-pair. 

Bird : Fly 

(A) Cat : Kitten 

(B) Fox : Vixen 

(C) Cattle : Herd 

(D) Hors 

E Gallop 

Answer: (D) 

Explanation: 

Answer marked by your team is incorrect. 

 

Q. 16 Select the number that DOES NOT belong to the following group. 

1, 2, 6, 21, 88, 446, 2676 

(A)

(B) 446 

(C) 21 

(D) 88 

Answer: (B) 

Explanation: 

1, 2, 6, 21, 88, 446, 2676…. 

1 × 1 + 1 = 2 , 

2 × 2 + 2 = 6 , 

6 × 3 + 3 = 21, 

21 × 4 + 4 = 88, 

88 × 5 + 5 = 445

(but in question 446 is given, hence 446 is wrong number series) 

 

Q. 17 Seven students are standing in a row. B is standing to the left of C and to the right of A. D is standing to the right of E and to the left of A. F is standing to the right of C and to the left of G. Who is standing in the middle? 

(A)

(B)

(C)

(D)

Answer: (D) 

 

Q. 18 Select the Venn diagram that best illustrates the relationship between the three given classes. 

Carrot, Vegetables, Meat 

Answer: (D) 

 

Q. 19 In the following fraction series, two fractions have been put within brackets. Study the series carefully and select the correct alternative with respect to the series. 

(A) Both the bracketed fractions are incorrect. 

(B) The first bracketed fraction (from the left) is correct and the second is incorrect. 

(C) The first bracketed fraction (from the left) is incorrect and the second is correct. 

(D) Both the bracketed fractions are correct. 

Answer: (D) 

 

Q. 20 Select the term that will come next in the following series. 

BMO, EOQ, HQS, KSU, ……….. 

(A) MTV 

(B) NUW 

(C) NTV 

(D) NVW 

Answer: (B) 

 

Q. 21 Identify the odd one from the following. 

(A) kmn 

(B) prt 

(C) bdf 

(D) suw 

Answer: (A) 

Explanation: 

p+2=r, r+2=t 

b+2=d, d+2=f 

s+2=u, u+2=w 

But option (A) does not follow the sequence, 

k+2=m, m+1=n (it should be M+2=O) 

 

Q. 22 Four pairs of words have been given, out of which three are alike in some manner, while one is different. Choose out the odd one. 

(A) Distance : Odometer 

(B) Wind : Taseometer 

(C) Current : Ammeter 

(D) Humidity : Hygrometer 

Answer: (B) 

Explanation: 

Wind is measured in anemometer. Hence Option (B) is Odd and incorrect. 

Remaining all other options measurement of instrument is correct form e.g.Humidity : Hygrometer, Current : Ammeter, Distance : Odometer. 

 

Q. 23 Select the figure that will come next in the following series. 

Answer: (D) 

 

Q. 24 If E is denoted by F, D is denoted by B, R is denoted by K. U is denoted by V, T is denoted by I,  A is denoted by C, G is denoted by X and H is denoted by W. then how will the word DAUGHTER be written? 

(A) BCWXVIFK 

(B) BCWXWFIK 

(C) BCVXWIKF 

(D) BCVXWIFK 

Answer: (D) 

Explanation: 

As given in Question, E=F, D=B, R=K. U=V, T=I, A=C, G=X and H=W 

Lets replace all values with above alphabets, DAUGHTER=BCVXWIFK 

 

Q. 25 Select the correct option to complete the following pattern. 

Answer: (B) 

General knowledge 

Instructions 

For the following questions answer them individually 

Q. 26 Which of the following is a contagious disease? 

(A) Diabetes 

(B) Stroke 

(C) Tuberculosis 

(D) Asthma 

Answer: (C) 

 

Q. 27 In which year was the constitution of Jammu and Kashmir adopted? 

(A) 1954 

(B) 1950 

(C) 1956 

(D) 1958 

Answer: (C) 

 

Q. 28 A group of fish is called a: 

(A) school 

(B) army 

(C) standard 

(D) bus 

Answer: (A) 

 

Q. 29 Inspired by the constitution of which nation has India added the feature of ‘Directive Principles’ in its constitution? 

(A) USA 

(B) Germany 

(C) Ireland 

(D) Russia 

Answer: (C) 

 

Q. 30 On which day is the World Population Day celebrated? 

(A) 2nd june

(B) 15th August

(C) 5th September

(D) 11th July

Answer: (D) 

 

Q. 31 What was the approximate distance that Mahatma Gandhi marched for the Salt Satyagraha Movement? 

(A) 241 miles 

(B) 350 miles 

(C) 400 miles 

(D) 123 miles 

Answer: (A) 

 

Q. 32 The Bretton Woods Agreementled to the creation of which one of the following institutions? 

(A) United Nations 

(B) United Nations Educational, Scientific and Cultural Organization 

(C) International Monetary Fund 

(D) International Labour Organization 

Answer: (C) 

 

Q. 33 What is the national sport of China? 

(A) Cricket 

(B) Football 

(C) Rugby 

(D) Table Tennis 

Answer: (D) 

 

Q. 34 What is the first age in the prehistoric period called? 

(A) Palaeolithic age 

(B) Metal age 

(C) Copper age 

(D) Neolithic age 

Answer: (A) 

 

Q. 35 On which date is the International Mountain Day celebrated? 

(A) 14th November 

(B) 11th December

(C) 2nd October 

(D) 13th January

Answer: (B) 

 

Q. 36 Who is the author of the book ‘Flight of Pigeon’? 

(A) Shobha Dey 

(B) Gita Piramal 

(C) Sudha Murthy 

(D) Ruskin Bond 

Answer: (D) 

 

Q. 37 The run-and-touch game of ‘Kho Kho’ originated in which Indian state? 

(A) Kerala 

(B) Jammu and Kashmir 

(C) Goa 

(D) Maharashtra 

Answer: (D) 

 

Q. 38 The Living Root Bridges are a rare ecological phenomenon seen in which state of India? 

(A) West Bengal 

(B) Meghalaya 

(C) Sikkim 

(D) Mizoram. 

Answer: (B) 

 

Q. 39 The yolk of an egg contains more than 90% of which of the given vitamins? 

(A) Vitamin C 

(B) Vitamin B12 

(C) Vitamin K 

(D) Vitamin (A) 

Answer: (B) 

 

Q. 40 A seesaw is an example of which concept of physics? 

(A) Current 

(B) Time 

(C) Electricity 

(D) Force 

Answer: (D) 

 

Q. 41 While trying to promote growth by tax cuts as the fiscal policy, which type of policy is the government trying to follow? 

(A) Expansionary 

(B) Contractionary 

(C) Easy money 

(D) Tight money 

Answer: (A) 

 

Q. 42 Who among the following has been conferred the Rajiv Gandhi National Sadbhavana Award 2018? 

(A) Anna Hazare 

(B) Medha Patkar 

(C) Kailash Satyarthi 

(D) Gopalkrishna Gandhi 

Answer: (D) 

 

Q. 43 Urs Festival in Ajmer commemorates the death anniversary of which Sufi Saint? 

(A) Khwaja Moinuddin Chisti 

(B) Hazarat Nizamuddin 

(C) Mian Mir 

(D) Salim Chishti 

Answer: (A) 

 

Q. 44 ‘Kunbi Dance’ is a popular dance associated with which Indian state? 

(A) Mizoram 

(B) Sikkim 

(C) Jammu and Kashmir 

(D) Goa 

Answer: (D) 

 

Q. 45 Who was the legendary co-founder and retired chairman of Indian tech giant Infosys? 

(A) Ashish Goel 

(B) NR Narayan Murthy 

(C) Yashish Dahiya 

(D) Navneet Singh 

Answer: (B) 

 

Q. 46 What do we call the stock of food grains purchased by the Government of India from the farmers through the Food Corporation of India (FCI)? 

(A) Incentivized stock 

(B) Return stock 

(C) Subsidized stock 

(D) Buffer stock 

Answer: (D) 

 

Q. 47 In which year did the French traveller Francois Bernier arrive in India? 

(A) 1656 

(B) 1658 

(C) 1678 

(D) 1662 

Answer: (B) 

 

Q. 48 What does the 103rd constitutional amendment deal with? 

(A) Women’s reservation 

(B) Economic reservation 

(C) Right to education 

(D) Right against exploitation 

Answer: (B) 

 

Q. 49 In which year did Emperor Akbar build the Buland Darwaja? 

(A) 1502 

(B) 1526 

(C) 1534 

(D) 1601 

Answer: (D) 

 

Q. 50 Which of the following is the longest river in Europe? 

(A) Mekong 

(B) Zambezi 

(C) Sepik 

(D) Volga 

Answer: (D) 

Quant 

Instructions 

For the following questions answer them individually 

Q. 51 A, B and C can do a piece of work in 20, 40 and 80 days respectively. In how many days can A do the work if he is assisted by B and C on every third day? 

(A) 7 days 

(B) 15.25 days 

(C) 18.25 days 

(D) 16.25 days 

Answer: (D) 

Explanation: 

(A) can do a work in 20 days 

(B) can do a work in 40 days 

(C) can do a work in 80 days 

LCM of 80,40 and 20 is 80 units 

Each day (A) does 80/20 =4 units of work 

Each day (B) does 80/40 =2 units of work 

Each day (C) does 80/80 =1 unit of work 

In first three days total wok=4+4+4+2+1=15 units 

For 15 days total work=15*5=75 units of work is one 

On 16th day 4 units is done and on 17th day (1/4)th day 

So total 16.25 days 

 

Q. 52 Case A. In a certain store, the profit is 340% of the cost. 

Case B. If the cost increases by 32% but the selling price remains constant, approximately what percentage of the selling price is the profit in case A? 

(A) 100% 

(B) 32% 

(C) 70% 

(D) 320% 

Answer: (C) 

 

Q. 53 A circle, with radius 8 cm, which has the area equal to the area of a triangle with base 8 cm. Then the length of the corresponding altitude of triangle is: 

(A) 38 π cm 

(B) 16 π cm 

(C) 8 π cm 

(D) 18 π cm 

Answer: (B) 

 

Q. 54 The captain of a cricket team of 11 members is 28 years old and the wicket keeper is 4 years older than him. If the ages of these two are excluded, the average age of the remaining players is two years less than the average age of the whole team. What is the average age of the team? 

(A) 15 years 

(B) 22 years 

(C) 21 years 

(D) 29 years 

Answer: (C) 

Explanation: 

Age of the captain=28 years 

Age of the wicket keeper=32 years 

let the sum of other 9 players =s 

Therefore s/9 =((s+28+32)/11)-2 

(s/9)=((s+60)/11) -2 

s/9 =(s+38)/11 

11s=9s+342 

2s=342 

s=171 

Total sum of ages of team=171+60=231 

Average=231/11 

=21 

 

Q. 55 Two numbers are respectively 25% and 50% more than a third number. The ratio of the first number to second number is: 

(A) 1 : 2 

(B) 2 : 1 

(C) 6 : 5 

(D) 5 : 6 

Answer: (D) 

Explanation: 

Let the third number be x 

then the first number will be 1.25x 

and second number will be 1.50x 

Ratio=1.25x/1.50x 

=5/6 

 

Q. 56 (42+4) / 5 + 5 X 2 – 3 x 4 is: 

(A)

(B)

(C)

(D)

Answer: (D) 

 

Q. 57 A hollow iron pipe is 28 cm long and its external diameter is 10 cm. If the thickness of the pipe is 2 cm and iron weighs 5 g/cm3, then the weight of the pipe is: 

(A) 70.4 kg 

(B) 7.04 kg 

(C) 74 kg 

(D) 7.40 kg 

Answer: (B) 

 

Q. 58 A train can travel 50% faster than a car. Both start from point A at the same time and reach point B, 60 km away from point A, at the same time. On the way, however, the train lost about 12.5 min while stopping at the stations. The speed of the car is: 

(A) 130 km/h 

(B) 100 km/h 

(C) 110 km/h 

(D) 96 km/h 

Answer: (D) 

Explanation: 

Let the speed of the car =S 

Then speed of the train =3S/2 

They travel same distance and reach at the same point but due to stoppages train looses 12.5 minutes time taken for car to travel 60 km=60/S 

Therefore (60/(3S/2))+12.5/60=(60/S) 

40/S + 12.5/60=60/S 

20/S=5/24 

S=24*4 

S=96 km/hr 

 

Q. 59 The following pie-chart shows the percentage distribution of the expenditure incurred in publishing a book. Study the pie-chart and answer the questions based on it. 

Royalty on the book is less than the paper cost by: 

(A) 25% 

(B) 30% 

(C) 50% 

(D) 40% 

Answer: (D) 

Explanation: 

Given expenditure on royalty=15% 

Expenditure on paper cost=25% 

Required percentage=((25-15)/25)*100 

=10*100/25 

=40% 

 

Q. 60 If the mean of the observations x, x + 4, x + 5, x + 7, x + 9 is 9, then the mean of the last three observations is: 

(A)

(B) 10 

(C) 11 

(D)

Answer: (C) 

Explanation: 

Sum =x+ x + 4+ x + 5+ x + 7+ x + 9 

=5x+25 

Mean =(5x+25)/5 

x+5=9 

x=4 

Last three observations are 9,11 and 13 

mean=33/3 

=11 

 

Q. 61 The following pie-chart shows the percentage distribution of the expenditure incurred in publishing a book. Study the pie-chart and answer the questions based on it. 

What is the central angle of the sector corresponding to the expenditure incurred on binding? 

(A) 72 

(B) 90 

(C) 48 

(D) 70 

Answer: (A) 

Explanation: 

Percent of expenditure on binding is given as 20% in the pie diagram Therefore 100%=360 

20%=360*20/100 

=72 degrees 

 

Q. 62 A family consists of two grandparents, three parents and four grandchildren. The average age of the grand parents is 65 years, that of the parents is 32 years and that of the grand children is 8 years. What is the average age of the family? 

(A) None of These 

(B) 28⅔ Years

(C) 28 Years

(D) 28⅓ Years

Answer: (B) 

Explanation: 

Given 2 grandparents average as 65 so sum of their ages=65*2=130 years 

Also given 3 parents average as 32 so sum of their ages=32*3=96 years 

Also given 4 grandchildren average as 8 so sum of their ages=8*4=32 years 

Total sum of ages=130+96+32=258 years 

Average=258/9 

=86/3 years 

 

Q. 63 A person has planned to sell his old type writer for ₹4500, he would lose 10%. To gain 20% of profit, he should sell it for: 

(A) ₹6000 

(B) ₹7000 

(C) ₹7200 

(D) ₹8500 

Answer: (A) 

Explanation: 

Given selling price=Rs 4500 

loss percent=10% 

SP=0.9 CP 

4500=0.9CP 

CP=4500/0.9 

CP=Rs 5000 

Given he wants to get 20% profit 

Therefore SP=1.2 CP 

SP=1.2*5000 

SP=Rs 6000 

 

Q. 64 Salaries of Ravi and Sumit are in the ratio 2:3. If the salary of each is increased by ₹3000, the new ratio becomes 35:50. What is Sumit’s new salary? 

(A) ₹18,000 

(B) ₹21,000 

(C) ₹27,000 

(D) ₹30,000 

Answer: (D) 

Explanation: 

Let the salaries of ravi and sumit be 2x and 3x 

salaries increased by 3000 

new salaries are 2x+3000 an 3x+3000 

ratio =7:10 

(2x+3000)/(3x+3000)=7/10 

20x+30000=21x+21000 

x=9000 

Sumit’s new salary=3(9000)+3000 

=30000 

 

Q. 65 The compound interest on ₹40,000 at 6% per annum is ₹4,944. What is the period (in years) for which the amount is invested? 

(A) 5 years 

(B) 4 years 

(C) 3 years 

(D) 2 years 

Answer: (D) 

 

Q. 66 A hall is 15 m long and 12 m broad.If the sum of the areas of the floor and the ceiling is equal to the sum of the areas of four walls, m3 the volume(in ) of the hall is: 

(A) 1600 

(B) 900 

(C) 1200 

(D) 720 

Answer: (C) 

Explanation: 

Area of the floor and ceiling will be same and it is equal 

i.e l*b=15*12=180 sq m 

Area of four walls will be 2*l*h+2*b*h=2h(l+b) 

2h(l+b)=360 

h(12+15)=180 

h=180/27 

h=20/3 

Volume=l*b*h=12*15*20/3 

=1200 cubic meters 

 

Q. 67 If a person had walked at the speed of 15 km/h instead of 10 km/h, he would have walked 25 km more. The actual distance travelled by him was: 

(A) 70 km 

(B) 56 km 

(C) 50 km 

(D) 80 km 

Answer: (C) 

Explanation: 

In both the cases time taken ‘t’ is same. 

Therefore 10t=15t-25 

15t-10t=25 

5t=25 

t=5 hours 

Initial speed was 10 km/hr 

and so distance traveled=10*5 

=50 km 

 

Q. 68 Raman paid ₹13,300 as simple interest after 9 years. He had borrowed some money at the rate of 6% for first two years, same money at 9% for next three years and at 14% for rest of the period. How much money(in ₹) did he borrow? 

(A) 10,000 

(B) 14,000 

(C) 12,500 

(D) 12,000 

Answer: (B) 

Explanation: 

We know I=PTR/100 

let the money borrowed be ‘x’ 

then for first two years it is 6% so interest=0.12x 

for next 3 years it is 9% so interest=0.27x 

for next four years it is 14% so interest =0.56x 

Total interest=0.12x+0.27x+0.56 

=0.95x 

therefore 0.95x=13300 

x=13300/0.95 

x=14000 

 

Q. 69 If 15 workers can earn ₹1,800 in 10 days. Find the earning (in ₹) of 5 workers in 8 days. 

(A) 400 

(B) 360 

(C) 540 

(D) 480 

Answer: (D) 

Explanation: 

Given 15 workers 10 days earn Rs 1800 

150 workdays=Rs 1800 

40 workdays=1800*40/150 

=40*12 

=Rs 480 

 

Q. 70 The following pie-chart shows the percentage distribution of the expenditure incurred in publishing a book. Study the pie-chart and answer the questions based on it. 

If for a certain quantity of books, the publisher has to pay ₹30,600 as printing cost, then what will be amount of paper cost to be paid for these books? 

(A) ₹28,250 

(B) ₹38,250 

(C) ₹38,000 

(D) ₹30,2503 

Answer: (B) 

Explanation: 

Given in the pie diagram printing cost=20% of total expenses 

paper cost=25% of total expenses 

Therefore 20%=30600 

25%=25*30600/20 

=7650*5 

=38250 

 

Q. 71 Find the greatest number that will divide 47, 95 and 187 so as to leave the same remainder in each case. 

(A)

(B) 13 

(C)

(D)

Answer: (D) 

Explanation: 

In each case they leave the same remainder and so the HCF of differences between the numbers has to be taken Difference 1=95-47=48 

Difference 2=187-47=140 

Difference 1=187-95=92 

HCF of 48,140 and 92 

48=2*2*2*2*3 

140=2*2*5*7 

92=2*2*23 

HCF=4 

 

Q. 72 Raj got a new chair at 35% discount. Had Raj got no discount, Raj would have had to pay ₹238 more. How much did Raj pay for the chair? 

(A) ₹452 

(B) ₹442 

(C) ₹416 

(D) ₹424 

Answer: (B) 

Explanation: 

Let the cost price be x 

He got 35% discount and so he paid 0.55x but given if no discount then he would have paid 238 more and so 0.35x=238 

x=680 

So Raj paid 680-238 

=Rs 442 

 

Q. 73 In a mixture of 60 L, the ratio of milk and water is 2:3, then the quantity of water in the mixture is: 

(A) 20 L 

(B) 45 L 

(C) 36 L 

(D) 16 L 

Answer: (C) 

 

Q. 74 What percentage of numbers from 1 to 80 have 1 or 9 in the units digit? 

(A) 1% 

(B) 20% 

(C) 21% 

(D) 14% 

Answer: (B) 

Explanation: 

From 1 to 9 each of 1 and 9 comes once at the units digit i.e 2 times(1 or 9) From 10 to 19 each of 1 and 9 comes once at the units digit i.e 2 times Similarly till 80 they come once for every ten numbers and so Total=8*2=16 times 

Total numbers =80 

Percentage=((16)/(80))*100 

=20% 

 

Q. 75 Simplify: 

8.65 − [4 + 0.5 of (8.8 − 2.3 × 3.5)] 

(A) 4.725 

(B) 4.275 

(C) 3.275 

(D) 4.527 

Answer: (B) 

Explanation: 

8.65 − [4 + 0.5 of (8.8 − 2.3 × 3.5)] 

= 8.65 − [4 + 0.5 of (8.8 − 8.05)] 

= 8.65 − [4 + 0.5 of (0.75)] 

= 8.65 − [35/8] 

= =8.65-4.375 

= 4.275 

English 

Instructions 

For the following questions answer them individually 

Q. 76 Select the word which means the same as the group of words given. 

Someone in love with himself. 

(A) Philologist 

(B) Humanist 

(C) Narcissist 

(D) Philanthropist 

Answer: (C) 

 

Q. 77 From the given options, identify the segment in the sentence which contains the grammatical error. The book provide ample guidance and practice in learning English conversations effectively for all age groups. 

(A) The book provide 

(B) for all age groups 

(C) ample guidance and practice 

(D) in learning English conversations 

Answer: (A) 

 

Q. 78 Select the most appropriate option to fill in the blank. 

Sometimes a person’s intelligence is not fully appreciated ………….. he/she gets an opportunity. 

(A) upto 

(B) later 

(C) until 

(D) unless 

Answer: (C) 

 

Q. 79 Select the most appropriate option to substitute the underlined segment in the given sentence. If there is no need to substitute it, select No improvement. 

Passengers are requested to go for security check before they proceed to board the flight. 

(A) are request to go for 

(B) are requests to go for 

(C) are requesting to go for 

(D) No improvement 

Answer: (D) 

 

Q. 80 Select the most appropriate option to substitute the underlined segment in the given sentence. If there is no need to substitute it, select No improvement. 

My neighbour along with his children is going tonight to watch a movie based over the life of martyrs . 

(A) a movie based on the life of martyrs 

(B) No improvement 

(C) a movie basing on life of martyrs 

(D) a movie base on the life of martyrs 

Answer: (A) 

Instructions 

In the following passage some words have been deleted. Fill in the blanks with the help of the alternatives given. Passage: 

Obesity can be (1)………. by eating healthy food and increasing physical activity. There is no need to alter the quantity of the food but the quality of the food must be (2)………….. and improvised. Avoiding red meat, oily food, extra fats in form of ghee, butter etc. can dramatically help to reduce the weight. Moreover, if the right eating is accompanied(3)………….. physical work out, it becomes the perfect way to lose weight and stay healthy. In some patients (4)…………. is also carried out. The choice of the treatment(5)…………. on various factors and the decision is taken by the physician by thoroughly examining the patient. 

 

Q. 81 Select the most appropriate option that will fill in the blank number 1. 

(A) magnified 

(B) destroyed 

(C) tolerated 

(D) managed 

Answer: (D) 

 

Q. 82 Select the most appropriate option that will fill in the blank number 2. 

(A) checked 

(B) decreased 

(C) distinguished 

(D) satisfied 

Answer: (A) 

 

Q. 83 Select the most appropriate option that will fill in the blank number 3. 

(A) with 

(B) of 

(C) at 

(D) for 

Answer: (A) 

 

Q. 84 Select the most appropriate option that will fill in the blank number 4. 

(A) amputation 

(B) surgery 

(C) segmentation 

(D) dissection 

Answer: (B) 

 

Q. 85 Select the most appropriate option that will fill in the blank number 5. 

(A) avoids 

(B) depends 

(C) ignores 

(D) loathes 

Answer: (B) 

Instructions 

For the following questions answer them individually 

 

Q. 86 Select the option that means the same as the given idiom. 

To pull oneself together 

(A) To hide important facts and reasons 

(B) To put necessary matters on the table 

(C) To keep working constantly with attention 

(D) To calm oneself and begin to think or act 

Answer: (D) 

 

Q. 87 Select the correctly spelt word. 

(A) Rehearsal 

(B) Rehersal 

(C) Reehersal 

(D) Rehearsel 

Answer: (A) 

 

Q. 88 Select the option that means the same as the given idiom. 

To be fair and square 

(A) To be fair in colour 

(B) To be honest 

(C) To be good at drawing 

(D) To be in a good physique 

Answer: (B) 

 

Q. 89 Select the most appropriate option to substitute the underlined segment in the given sentence. If there is no need to substitute it, select No improvement. 

No sooner he has returned to home than his boss called him for an urgent meeting. 

(A) No sooner had he returned home 

(B) No sooner he returned to home 

(C) No sooner he have returned to home 

(D) No improvement 

Answer: (A) 

 

Q. 90 Select the antonym of the given word. 

EXTRAVAGANT 

(A) Economical 

(B) Factual 

(C) Untrue 

(D) Wonderful 

Answer: (A) 

 

Q. 91 Select the most appropriate word to fill in the blank. 

A person’s individual risk for many diseases most often ……… from a combination of genes, environment and lifestyle. 

(A) stems 

(B) checks 

(C) diagnoses 

(D) stops 

Answer: (A) 

 

Q. 92 Select the most appropriate option to fill in the blank. 

People can only form new habits when they ………. particular patterns of behaviour. 

(A) has adopted 

(B) having adopted 

(C) had adopted 

(D) have adopted 

Answer: (D) 

 

Q. 93 From the given options, identify the segment in the sentence which contains the grammatical error. Taking the help of people with rich experience of life will saves you much time and effort. 

(A) Taking the help of 

(B) much time and effort 

(C) people with rich experience 

(D) will saves you 

Answer: (D) 

 

Q. 94 Select the synonym of the given word. 

STRAY 

(A) Slight 

(B) Wander 

(C) Steady 

(D) Gruesome 

Answer: (B) 

 

Q. 95 From the given options, identify the segment in the sentence which contains the grammatical error. Just because you are learning a language does not mean you need to limiting your thinking. 

(A) does not mean 

(B) need to limiting your thinking 

(C) Just because 

(D) you are learning a language 

Answer: (B) 

 

Q. 96 Select the most appropriate word to fill in the blank. 

This is a good time for all of us to a new year’s party. 

(A) collect 

(B) initiate 

(C) invite 

(D) organize 

Answer: (D) 

 

Q. 97 Select the word which means the same as the group of words given. 

(A) large number of fish swimming together. 

(A) Herd 

(B) Pack 

(C) Mob 

(D) Shoal 

Answer: (D) 

 

Q. 98 Select the synonym of the given word. 

ADULTERATE(D) 

(A) Combined 

(B) Fused 

(C) Contaminated 

(D) Concentrated 

Answer: (C) 

 

Q. 99 Select the correctly spelt word. 

(A) Quantetative 

(B) Quantitativ 

(C) Quantitetive 

(D) Quantitative 

Answer: (D) 

 

Q. 100 Select the antonym of the given word. 

BENIGN 

(A) Hostile 

(B) Amiable 

(C) Mild 

(D) Genial 

Answer: (A) 

SSC GD 1 March 2019 Shift-I Previous Year Paper

SSC GD 1st March 2019 Shift-1 

Reasoning 

Instructions 

For the following questions answer them individually 

Q. 1 Select the option that is related to the third term in the same way as the second term is related to the first term. 

Where : Place :: When : ……… 

(A) There 

(B) Then 

(C) Why 

(D) Time 

Answer: D 

 

Q. 2 Six friends, P, Q, R, S, T and U, are sitting in a closed circle facing the centre such that each one is facing another one. P is facing S. R is between P and Q. U is between P and T who is sitting right of S. Who is sitting to the immediate right of R? 

(A)

(B)

(C)

(D)

Answer: C 

Explanation: 

Six friends, P, Q, R, S, T and U, are sitting in a closed circle facing the centre such that each one is facing another one. P is facing S. U is between P and T who is sitting right of S. R is between P and Q. 

Q is sitting to the immediate right of R. 

Option (C) is correct. 

 

Q. 3 Read the given statements and conclusion carefully and decide which of the following options is true with respect to the given conclusion. 

Statements: 

The Gateway of India is in Mumbai. 

Mumbai is in India. 

Conclusion: 

The Gateway of India is in India. 

(A) Probably false 

(B) True 

(C) False 

(D) Probably true 

Answer: B 

 

Q. 4 A square transparent sheet with a pattern is given. Select from among the four alternatives how the pattern would appear when the transparent sheet is folded at the dotted line. 

Answer: B 

Explanation: If the transparent sheet with the following pattern is folded along the dotted lines, 

the following pattern appears, 

Option (B) is correct. 

 

Q. 5 Select the fraction that will next in the following series. 

(A) 14/15

(B) 14/16

(C) 13/15

(D) 13/14

Answer: A 

 

Q. 6 Select the Venn diagram that best illustrates the relationship between the three given classes. 

Flowers, Red, Clothes 

Answer: B 

 

Q. 7 U,V, W, X, Y and Z are seated in a circle facing the centre such that each one is facing another one. X is between Z and V. U is second to the left of X and second to the right of Y. Who is facing X? 

(A)

(B)

(C)

(D)

Answer: A 

Explanation: 

U,V, W, X, Y and Z are seated in a circle facing the centre such that each one is facing another one. 

X is between Z and V. U is second to the left of X and second to the right of Y. 

W is facing X. 

 

Q. 8 Select the term that will come in the following series. 

PC, RF, TI, VL, XO, ……… 

(A) ZQ 

(B) ZR 

(C) YQ 

(D) YR 

Answer: B 

 

Q. 9 In the following number series, two numbers have been put within brackets. Study the series carefully and select the correct alternative with respect to the series. 

1, 1, 2, 6, 24, (121), 720, (5040), 40320 

(A) The first bracketed number (from the left) is correct and the second is incorrect. 

(B) The first bracketed number (from the left) is incorrect and the second is correct. 

(C) Both the bracketed numbers are in correct. 

(D) Both the bracketed numbers are correct. 

Answer: B 

Explanation: 

1 × 1 = 1 

1 × 2 = 2 

2 × 3 = 6 

6 × 4 = 24 

24 × 5 = 120 

120 × 6 = 720 

720 × 7 = 5040 

5040 × 8 = 40320 

Option (B) is correct. 

 

Q. 10 If D = 3, G = 6, and FUN = 38, then PRESIDENT = ………. 

(A) 119 

(B) 110 

(C) 100 

(D) 101 

Answer: D 

 

Q. 11 If SQUARE is coded as TSXEWK, then TRIANGLE will be coded as ………. 

(A) UTLFSMSM 

(B) UTLESNSM 

(C) UTLESMSM 

(D) UTLESMSN 

Answer: C 

 

Q. 12 Select the word-pair that is analogous to the given word-pair. 

France : Paris 

(A) Iran: Rial 

(B) India : Mumbai 

(C) Cuba : Havana 

(D) China : Yuan 

Answer: C 

Q. 13 Select the option in which the given figure is embedded. 

Answer: A 

 

Q. 14 Select the number that DOES NOT belong to the following group. 

3, 6, 9, 12, 15, 18, 21, 24, 28, 30 

(A) 28 

(B) 30 

(C) 18 

(D) 15 

Answer: A 

 

Q. 15 Select the number-pair that is different from the rest. 

(A) 7-345 

(B) 5-125 

(C) 9-729 

(D) 3-27 

Answer: A 

 

Q. 16 Choose the option that most closely resembles the mirror image of the given word when mirror is placed at right side. 

AIM 

(A) AIM 

(B) MIA

(C) AIW 

(D) WIA 

Answer: B 

 

Q. 17 Four pairs of words have been given, out of which three are alike in some manner, while one is different. Choose out the odd one. 

(A) Brother : Sister 

(B) Lion : Lioness 

(C) Bear : Cub 

(D) Cock : Hen 

Answer: C 

 

Q. 18 The statements below are followed by three conclusions labeled I, II and III. Assuming that the information in the statements is true, even if it appears to be at variance with generally established facts, decide which conclusion(s) logically and definitely follow(s) from the information given in the statements . 

Statements: 

All balls are boats. 

All boats are cars. 

Conclusions: 

I. Some cars are balls. 

II. All balls are cars. 

III. All cars are balls. 

(A) All the conclusions follow. 

(B) Only conclusion III follows. 

(C) Only conclusions II and III follow. 

(D) Only conclusions I and II follow. 

Answer: D 

 

Q. 19 Select the correct option to complete the following pattern. 

Answer: D 

 

Q. 20 Which two signs need to be interchanged to make the given equation correct? 

20 ÷ 10 × 100 − 10 + 100 = 110 

(A) + and÷ 

(B) × and − 

(C) ÷ and × 

(D) + and − 

Answer: D 

 

Q. 21 Select the figure that will come next in the following series. 

Answer: A 

 

Q. 22 Diya have five subjects in her academics namely A, B, C, D and E. She studies them from Monday to Friday of a week. On each day, only one subject will be studied. D or E should not be either the first or last to be studied. E should be immediately followed by C. B should be studied immediately after D. One subject will be studied between A and B. 

Then which subject she immediately reads after Monday? 

(A)

(B)

(C)

(D)

Answer: C 

 

Q. 23 Four words have been given out of which three are alike in some manner, while one is different. Choose the odd one. 

(A) Volume 

(B) Force 

(C) Volt 

(D) Power 

Answer: C 

 

Q. 24 Select the option that is related to the third number in the same way as the second numberis related to the first number. 

9 : 100 :: 13 : …….. 

(A) 130 

(B) 144 

(C) 196 

(D) 169 

Answer: C 

Explanation: 

9 : (9 + 1)2 :: 13 : (13 + 1)2 

9 : 100 :: 13 : 196 

Option (C) is correct 

 

Q. 25 Identify the odd one from the following. 

(A) PRT 

(B) VXZ 

(C) HJL 

(D) CFG 

Answer: D 

Explanation: 

1st alphabet → (+2)2nd alphabet → (+2)3rd alphabet. 

Option (D) don’t follow the above pattern and is the odd one out 

Option (D) is correct. 

General knowledge 

Instructions 

For the following questions answer them individually 

Q. 26 ‘Tulsidas’, a poet who lived during the medieval times of Indian history, wrote “Ramcharitmanas’ in which of the following languages? 

(A) Pali 

(B) Prakrit 

(C) Awadhi 

(D) Sanskrit 

Answer: C 

 

Q. 27 What was the capital of the French colony in India? 

(A) Pondicherry 

(B) Calicut 

(C) Cochin 

(D) Goa 

Answer: A 

 

Q. 28 Which famous Indian personality is the founder of ‘Paani Foundation’? 

(A) Shah Rukh Khan 

(B) Salman Khan 

(C) Amir Khan 

(D) Ranveer Singh 

Answer: C 

 

Q. 29 With the commencement of the 7th century, who ascended the throne of Thaneshwar and Kannauj? 

(A) Krishnadeva 

(B) Harshavardhana 

(C) Rajendra Chola I 

(D) ChandraGupta II 

Answer: B 

 

Q. 30 Who Is the cofounder of IndiGo Airlines? 

(A) Kiran Mazumdar Shaw 

(B) Pawan Munjal 

(C) Y C Deveshwar 

(D) Rahul Bhatia 

Answer: D 

 

Q. 31 ‘Vijay Amvitraj’ was a legendary Indian player associated with which of the following sports? 

(A) Lawn tennis 

(B) Swimming 

(C) Rifle shooting 

(D) Badminton 

Answer: A 

 

Q. 32 What is India’s rank in the World Bank’s Human Capital Index (HCI) rankings as of Feb 2019? 

(A) 124 

(B) 115 

(C) 126 

(D) 104 

Answer: B 

 

Q. 33 ‘Talatal Ghar’ is a historical monument situated in which of the following Indian states? 

(A) Assam 

(B) West Bengal 

(C) Sikkim 

(D) Odisha 

Answer: A 

 

Q. 34 “Which among the following is the longest river in South India? 

(A) Narmada 

(B) Tungabhadra 

(C) Godavari 

(D) Krishna 

Answer: C 

 

Q. 35 Which disease among the given options spreads via pigs? 

(A) Nipah 

(B) Swine flu 

(C) Zika 

(D) Plague 

Answer: B 

 

Q. 36 Riboflavin present in fish is also known as: 

(A) Vitamin C

(B) Vitamin K 

(C) Vitamin B12 

(D) Vitamin B2 

Answer: D 

 

Q. 37 Which mountain pass in the world has the highest motorable road in the world? 

(A) Jelep-La 

(B) Nathu-La 

(C) Shipki-La 

(D) Dhungria-La 

Answer: D 

 

Q. 38 The production of a commodity through natural process is an activity associated to which sector of the Indian Economy? 

(A) Tertiary 

(B) Secondary 

(C) Quaternary 

(D) Primary 

Answer: D 

 

Q. 39 As on 2018, which is the world’s largest political party in terms of primary membership? 

(A) The Bharatiya Janata Party 

(B) The Conservative Party of UK 

(C) The Democratic Party of the US 

(D) National republican Party US 

Answer: A 

 

Q. 40 Fiscal policy is based on the economic theory given by which of the following economist? 

(A) John Maynard Keynes 

(B) Adam Smith 

(C) Milton Friedman 

(D) John Nash 

Answer: A 

 

Q. 41 ‘Theyyam’ is a popular dance form of which of the following states? 

(A) Karnataka 

(B) Kerala 

(C) Gujarat 

(D) Manipur 

Answer: B 

 

Q. 42 The National Food Security Act was passed in which of the following years? 

(A) 2011 

(B) 2013 

(C) 2014 

(D) 2012 

Answer: B 

 

Q. 43 The movement of a horse drawn carriage is an example of which principle of physics? 

(A) Newton’s third law of motion 

(B) Ohm’s law 

(C) Archimedes principle 

(D) Avogadro’s law 

Answer: A 

 

Q. 44 Which city will host the upcoming Winter Olympic Games scheduled in 2022? 

(A) Los Angeles 

(B) Paris 

(C) Tokyo 

(D) Beijing 

Answer: D 

 

Q. 45 The festival of ‘Hareli’ is celebrated in which of the following Indian states? 

(A) Odisha 

(B) Chhattisgarh 

(C) Punjab 

(D) Madhya Pradesh 

Answer: B 

 

Q. 46 The Consumer Protection Bill, 2018 has replaced the Consumer Protection Act of which of the given years? 

(A) 1981 

(B) 1986 

(C) 1991 

(D) 1978 

Answer: B 

 

Q. 47 ‘Azadirachta indica’ is the botanical name of which evergreentree foundin the Indian sub-continent? 

(A) Curry tree 

(B) Pine tree 

(C) Hibiscus tree 

(D) Margosa tree 

Answer: D 

 

Q. 48 ‘The Legend of Laxmi Prasad’ was the title of a book written by which one of the following authors? 

(A) Arundhati Roy 

(B) Twinkle Khanna 

(C) Jhumpa Lahiri 

(D) Shobha De 

Answer: B 

 

Q. 49 In which layer of the earth’s atmosphere is the ozone layer found? 

(A) Troposphere 

(B) Mesosphere 

(C) Exosphere 

(D) Stratosphere 

Answer: D 

 

Q. 50 As of January 2019, what is the number of articles in the Indian constitution? 

(A) 431 

(B) 402 

(C) 448 

(D) 395 

Answer: C 

Quant 

Instructions 

For the following questions answer them individually 

Q. 51 The Area of a rectangle is 27m2 and its length is 3 times of its breadth. The perimeter of the rectangle is: 

(A) 24cm 

(B) 12cm 

(C) 42cm 

(D) 28cm 

Answer: A 

 

Q. 52 Study the Table Properly and answer by interpreting the data. 

The table shows the percentage population of five districts in a state below poverty line and the proportion of males and females.

The total population in D1, is 30,000. The number of female under below poverty line in the district D1, is: 

(A) 2500 

(B) 3500 

(C) 2700 

(D) 1800 

Answer: C 

 

Q. 53 The value of 30 ÷ 6 × 5 of (2 + 3) − 12(3 × 2) is equal to: 

(A) 28 

(B) 35 

(C) 82 

(D) 53 

Answer: D 

 

Q. 54 Two numbers are less than a third number by 25 and 36 respectively. The percentage of first number less than second number is: 

(A) 61% 

(B) 11% 

(C) 18% 

(D) 52% 

Answer: D 

 

Q. 55 The value of 25 of 4 − 30 + (22 + 18) is: 

(A) 23 

(B) 84 

(C) 92 

(D) 37 

Answer: C 

 

Q. 56 A wheel makes 4000 revolution is covering a distance of 60km. The radius of the wheel is: 

(A) 8m 

(B) 8.25m 

(C) 4.68m 

(D) 2.39m 

Answer: D 

 

Q. 57 In class IX, the average of marks in science for six students was 48. After result declared, it was found in case of one student, the marks 45 were misread as 54. The correct average is : 

(A) 46.40 

(B) 46.50 

(C) 76.50 

(D) 64.39 

Answer: B 

 

Q. 58 Study the Table Properly and answer by interpreting the data 

The table shows the percentage population of five districts in a state below poverty line and the proportion of males and females.

If the total population of district D4, is 35000, then what is the number of females above the poverty line in the district D4? 

(A) 10,500 

(B) 14,240 

(C) 11,250 

(D) 12,450 

Answer: C 

 

Q. 59 In a class, the average age of 40 students is 12 years when teacher’s age is included to it, the average increases by 1. The age of teacher is : 

(A) 53 years 

(B) 51 years 

(C) 48 years 

(D) 68 years 

Answer: A 

 

Q. 60 Shopkeeper selling on article for ₹46 loses 8%. In order to gain of 6%, what should be the selling price of the article? 

(A) ₹65 

(B) ₹56 

(C) ₹53 

(D) ₹85 

Answer: C 

 

Q. 61 The areas of two squares are 16:9. The ratio of their perimeter is: 

(A) 9:16 

(B) 9:12 

(C) 12:16 

(D) 16:12 

Answer: D 

 

Q. 62 The difference between upper limit and lower limit of the class interval is called as: 

(A) Range 

(B) Mode 

(C) Frequency distribution 

(D) Class size 

Answer: D 

 

Q. 63 Study the Table Properly and answer by interpreting the data 

The table shows the percentage population of five districts in a state below poverty line and the proportion of males and females.

If the total population in the district D3 is 40,000, then what is the population of below poverty line in the district D3, ? 

(A) 10,000 

(B) 5,000 

(C) 8,000 

(D) 12,000 

Answer: C 

 

Q. 64 In how many years, a sum will be thrice of it at the rate of interest 5% per annum? 

(A) 25 years 

(B) 40 years 

(C) 30 years 

(D) 20 years 

Answer: B 

 

Q. 65 If (x + y) : (x − y) = 3 : 2 , then (x2 + y )2 : (x2 − y2) is in the ratio of: 

(A) 12:13 

(B) 5:12 

(C) 13:12 

(D) 24:5 

Answer: C 

 

Q. 66 In a university, the number of students studying science, mathematics and language are in the ratio of 2:4:9. If the number of students in Science, Mathematics and Language be increased 10%, 20% and 40% respectively. What will be the new ratio? 

(A) 12:23:63 

(B) 11:24:63 

(C) 24:11:63 

(D) 63:11:24 

Answer: B 

 

Q. 67 By Selling a TV for ₹8500 a person loses 15%. In order to gain of 5%, Selling price of TV should be: 

(A) ₹10,500 

(B) ₹10,000 

(C) ₹9,500 

(D) ₹9,000 

Answer: A 

Explanation: 

By Selling a TV for ₹8500 a person loses 15%. 

Therefore, CP = SP ÷ 0.85 = 8500 ÷ 0.85 = ₹10, 000 

In order to gain of 5%, Selling price of TV should be= 

SP = 1.05 × 10, 000 = ₹10, 500 

Option (A) is correct. 

 

Q. 68 ‘A’ pipe can empty a tank in 20 minutes. The second pipe ‘B’ has a diameter twice as that of ‘A’. If both A & B pipe are attached to the tank how much time will be required to empty the tank? 

(A) 10½ minutes 

(B) 25 minutes 

(C) 15 minutes 

(D) 6⅔ minutes 

Answer: D 

 

Q. 69 The HCF and LCM of two numbers are 12 and 240 respectively. If one of the number is 24, what is the another number 

(A) 120 

(B) 228 

(C) 264 

(D) 144 

Answer: A 

 

Q. 70 Rama and Hari can together finish a piece of work in 15 day. Rama works twice as fast as Hari, then Hari alone can finish work in : 

(A) 45 days 

(B) 30 days 

(C) 25 days 

(D) 20 days 

Answer: A 

 

Q. 71 Two numbers are 20% and 40% more than the third number respectively. The ratio of the first and second number is: 

(A) 7:6 

(B) 7:5 

(C) 6:7 

(D) 5:7 

Answer: C 

 

Q. 72 The compound interest on an amount for 2 years at 5% per annum compounded annually is ₹205. The simple interest is: 

(A) ₹250 

(B) ₹220 

(C) ₹200 

(D) ₹210 

Answer: C 

 

Q. 73 The distance between two stations is 500 km. (A) train starts from station ‘A’ at 9 am and travels towards station ‘B’ at 60 km/hrs. Another Train starts from station ‘B’ at 10 am and travels towards ‘A’ at 40 km/hrs. At what time both the train meet to each other. 

(A) 12.45 PM 

(B) 1.36 PM 

(C) 3.36 PM 

(D) 2.24 PM 

Answer: D 

 

Q. 74 (A) trader allows two successive discounts of 5% each on the marked price of a sofa set for ₹24,500. The selling price of the sofa set is: 

(A) ₹23,274.75 

(B) ₹23,275 

(C) ₹1,163.75 

(D) ₹22,111.25 

Answer: D 

 

Q. 75 A train is moving at 72 km/hrs. The distance covers in 15 minutes by the train is: 

(A) 18 km 

(B) 27 km 

(C) 36 km 

(D) 28 km 

Answer: A 

English 

Instructions 

For the following questions answer them individually 

Q. 76 Select the word which means the same as the group of words given. 

One who does a thing for pleasure and not as a profession 

(A) Sportsperson 

(B) Amateur 

(C) Volunteer 

(D) Athlete 

Answer: B 

 

Q. 77 Select the option that means the same as the given idiom. 

To hang together 

(A) To be over telephone with a friend 

(B) To keep holding on to something 

(C) to take a swing in a garden 

(D) To be connected with each other 

Answer: D 

 

Q. 78 Select the antonym of the given word. 

CONVENE 

(A) Disperse 

(B) Summon 

(C) Request 

(D) Gather 

Answer: A 

 

Q. 79 Select the most appropriate word to fill in the blank. 

You can master anything by the usual process of learning and practicing it ………….. 

(A) temporarily 

(B) occasionally 

(C) regularly 

(D) annually 

Answer: C 

 

Q. 80 Select the antonym of the given word. 

CONCUR 

(A) Contribute 

(B) Lead 

(C) Disagree 

(D) Distribute 

Answer: C 

 

Q. 81 Select the synonym of the given word. 

AFFINITY 

(A) Monotony 

(B) Beauty 

(C) Empathy 

(D) Decency 

Answer: C 

 

Q. 82 Select the most appropriate word to fill in the blank. 

Your parents may get …………… With your lifestyle and demand a change init. 

(A) fulfilled 

(B) pleased 

(C) satisfied 

(D) annoyed 

Answer: D 

 

Q. 83 Select the correctly spelt word. 

(A) Suplementary 

(B) Supplementary 

(C) Suppllementary 

(D) Supplementery 

Answer: B 

 

Q. 84 From the given options, identify the segment in the sentence which contains the grammatical error. While we would like all Indian children to go to school, we need to ponder why they does not. 

(A) why they does not 

(B) While we would like 

(C) Indian children to go to school 

(D) we need to ponder 

Answer: A 

 

Q. 85 Select the most appropriate option to fill in the blank. 

My forefinger is still ………… where I caught it in the door this morning. 

(A) burnt 

(B) scorched 

(C) bruised 

(D) damaged 

Answer: C 

 

Q. 86 Select the word which means the same as the group of words given. 

A person who is indifferent to the pains and pleasures of life. 

(A) Introvert 

(B) Social 

(C) Stoic 

(D) Solitary 

Answer: C 

 

Q. 87 Select the most appropriate option to substitute the underlined segment in the given sentence. If there is no need to substitute it, select No improvement. 

On the academic front you are likely to outshine others and take up a better career. 

(A) you were likely to outshine others 

(B) you should likely to outshine others 

(C) you are likely for outshine other 

(D) No improvement 

Answer: D 

 

Q. 88 Select the synonym of the given word. 

CUMBERSOME 

(A) Wonderful 

(B) Heavy 

(C) Graceful 

(D) Virtuous 

Answer: B 

Instructions 

In the following passage some words have been deleted. Fill in the blanks with the help of the alternatives given. Passage: 

“Success begins in the mind.” This is a wonderful quote to explain the power of having (1)………. in yourself. Since (2)………. wants success, they all try their best. Sometimes they will (3)……….. failure, but that’s what life is. Ups and downs are a part of it. All you need is to take that failure in your (4)…………. learn from it and never forget it. It doesn’t matter how or where you come from, success happens to those who (5)……….. hands with difficulties. 

Q. 89 Select the most appropriate option that will fill in the blank number 1. 

(A) doubt 

(B) disgust 

(C) faith 

(D) discard 

Answer: C 

 

Q. 90 Select the most appropriate option that will fill in the blank number 2. 

(A) everyone 

(B) each 

(C) someone 

(D) anyone 

Answer: A 

 

Q. 91 Select the most appropriate option that will fill in the blank number 3. 

(A) clash 

(B) encounter 

(C) see 

(D) oppose 

Answer: B 

 

Q. 92 Select the most appropriate option that will fill in the blank number 4. 

(A) run 

(B) stride 

(C) walk 

(D) wing 

Answer: B 

 

Q. 93 Select the most appropriate option that will fill in the blank number 5. 

(A) shake 

(B) see 

(C) move 

(D) avoid 

Answer: A 

Instructions 

For the following questions answer them individually 

 

Q. 94 Select the most appropriate option to substitute the underlined segment in the given sentence. If there is no need to substitute it, select No improvement. 

Those who are travelling today by train may have delayed in reaching their destinations. 

(A) travelling today in train may get delayed 

(B) No improvement 

(C) travel today by train may get delayed 

(D) travelling today by train may get delayed 

Answer: D 

 

Q. 95 Select the most appropriate option to substitute the underlined segment in the given sentence. If there is no need to substitute it, select No improvement. 

Statistical measure which track the money-saving behaviour haven’t improved much. 

(A) No improvement 

(B) measure which tracks money-saving behaviours 

(C) measures what track the money-saving behaviour 

(D) measures which track the money-saving behaviour 

Answer: D 

 

Q. 96 Select the correctly spelt word. 

(A) Badjet 

(B) Budget 

(C) Budjet 

(D) Bugett 

Answer: B 

 

Q. 97 From the given options, identify the segment in the sentence which contains the grammatical error. In India, the various practices and norms from bank’s transactions are laid down by the RBI only. 

(A) are laid down 

(B) norms from bank’s transactions 

(C) the various practices 

(D) by the RBI only 

Answer: B 

 

Q. 98 Select the option that means the same as the given idiom. 

Call it a day 

(A) Decide or agree to stop doing something 

(B) Attempting to do an impossible task 

(C) Disagreeing on everything someone is saying 

(D) Agreeing to reveal some secret information 

Answer: A 

 

Q. 99 From the given options, identify the segment in the sentence which contains the grammatical error. You are invite to build a better community by planting a seed of peace within and tree outside. 

(A) build a better community 

(B) within and a tree outside 

(C) You are invite to 

(D) by planting a seed of peace 

Answer: C 

 

Q. 100 Select the most appropriate option to fill in the blank. 

Whenever he refers to his friends he is pleased but when he talks of his foes he is ……….. 

(A) unhappy 

(B) possessed 

(C) serene 

(D) interested 

Answer: A 

×

Hello!

Click one of our representatives below to chat on WhatsApp or send us an email to info@vidhyarthidarpan.com

×